65: Critical Care

Réussis tes devoirs et examens dès maintenant avec Quizwiz!

A nurse is caring for a patient undergoing mechanical ventilation who is also receiving positive end-expiratory pressure (PEEP). What is the outcome that the nurse hopes to achieve with PEEP? 1 Expand collapsed alveoli 2 Decrease alveolar volume 3 Decrease bronchospasms 4 Prevent spontaneous breathing

1 Expand collapsed alveoli Positive end-expiratory pressure (PEEP) expands collapsed alveoli and improves resting lung volume by keeping the alveoli open and preventing them from collapsing during expiration. PEEP increases, not decreases, alveolar volume by keeping the alveoli expanded. PEEP has no direct effect on bronchospasms. PEEP allows, not prevents, spontaneous breathing of a patient undergoing mechanical respiration.

A feeding tube is placed in a patient receiving positive pressure ventilation to prevent inadequate nutrition. What should the nurse avoid while verifying the placement of the feeding tube? 1 Listening for air after injection 2 X-ray confirmation before initial use 3 Review of routine x-rays and aspirate 4 Marking and assessing the tube's exit site

1 Listening for air after injection While verifying the placement of feeding tube, the nurse should avoid listening for air after injection, because it is not a reliable method for verification of its placement. X-ray confirmation before initial use, review of routine x-rays, and marking and assessing the tube's exit site are all reliable methods for verifying the feeding tube placement.

List 4 indications for mechanical ventilation.

1. Apnea or impending inability to breathe 2. Acute respiratory failure 3. Severe hypoxia 4. Respiratory muscle fatigue

A patient is to be intubated for respiratory failure. Which factor indicates that tracheotomy would be preferable to endotracheal intubation? 1 The patient is at high risk for aspiration. 2 The patient is unable to clear secretions. 3 A long-term airway is probably necessary. 4 An upper airway obstruction is impairing the patient's ventilation.

3 A long-term airway is probably necessary. A tracheotomy is indicated when the need for an artificial airway is expected to be long term. Aspiration risk, an inability to clear secretions, and upper airway obstruction are indications for an artificial airway, but these are not specific indications for tracheotomy.

A patient is being mechanically ventilated. A high-pressure ventilation alarm sounds. The nurse should assess for what cause of this type of alarm? 1 Power failure 2 Insufficient gas flow 3 Condensate in tubing 4 Tracheotomy cuff leak

3 Condensate in tubing Presence of condensate or water in tubing triggers a high-pressure ventilation alarm. Power failure triggers ventilator inoperative or low battery alarm. Insufficient gas flow and tracheotomy cuff leak triggers low tidal volume or minute ventilation alarm.

25. When evaluating a patient with a central venous catheter, the nurse observes that the insertion site is red and tender to touch and the patient's temperature is 101.8° F. What should the nurse plan to do? a. Discontinue the catheter and culture the tip. b. Use the catheter only for fluid administration. c. Change the flush system and monitor the site. d. Check the site more frequently for any swelling.

ANS: A The information indicates that the patient has a local and systemic infection caused by the catheter, and the catheter should be discontinued to avoid further complications such as endocarditis. Changing the flush system, continued monitoring, or using the line for fluids will not help prevent or treat the infection.

32. A patient who is orally intubated and receiving mechanical ventilation is anxious and is "fighting" the ventilator. Which action should the nurse take next? a. Verbally coach the patient to breathe with the ventilator. b. Sedate the patient with the ordered PRN lorazepam (Ativan). c. Manually ventilate the patient with a bag-valve-mask device. d. Increase the rate for the ordered propofol (Diprivan) infusion.

ANS: A The initial response by the nurse should be to try to decrease the patient's anxiety by coaching the patient about how to coordinate respirations with the ventilator. The other actions may also be helpful if the verbal coaching is ineffective in reducing the patient's anxiety.

1. A patient's vital signs are pulse 90, respirations 24, and BP 128/64 mm Hg, and cardiac output is 4.7 L/min. The patient's stroke volume is _____ mL. (Round to the nearest whole number.)

ANS: 52 Stroke volume = Cardiac output/heart rate 52 mL = (4.7 L x 1000 mL/L)/90

The postanesthesia care unit (PACU) has several patients with endotracheal tubes. Which patient should receive the least amount of endotracheal suctioning? Transplantation of a kidney Replacement of aortic valve Cerebral aneurysm resection Formation of an ileal conduit

Cerebral aneurysm resection The nurse should avoid suctioning the patient after a craniotomy until it is necessary because suctioning will increase this patient's intracranial pressure. The patients with a kidney transplantation, aortic valve replacement, or formation of an ileal conduit will not be negatively affected by suctioning, although it should only be done when needed, not routinely.

1 Patients requiring ventilation up to 3 days are said to have received short-term ventilation. More than 3 days (7, 14, and 20 days) denotes long-term ventilation. Text Reference - p. 1625

The health care provider advises short-term ventilation for a patient. How many days would be considered short-term ventilation? 1 Up to 3 days 2 Up to 7 days 3 Up to 14 days 4 Up to 20 days

2 While caring for a patient requiring mechanical ventilation, the registered nurses (RNs) must administer sedatives by themselves; they should not delegate this task to unlicensed assistive personnel (UAP). UAP can be tasked with obtaining vital signs, measuring urine output, and performing bedside glucose tests. Text Reference - p. 1625

When caring for a critically ill patient on mechanical ventilation, what task must the registered nurses (RNs) perform by themselves and not delegate to unlicensed assistive personnel (UAP)? 1 Obtaining vital signs 2 Administering sedatives 3 Measuring urine output 4 Performing bedside glucose test

Which factor indicates that tracheotomy would be preferable to endotracheal intubation? A. The patient is unable to clear secretions. B. The patient is at high risk for aspiration. C. A long-term airway is probably necessary. D. An upper airway obstruction is impairing the patient's ventilation.

Ans. C A tracheotomy is indicated when the need for an artificial airway is expected to be long term. Aspiration risk, an inability to clear secretions, and upper airway obstruction are indications for an artificial airway, but these are not specific indications for tracheotomy.

A patient is being given a medication that stimulates her parasympathetic system. Following administration of this medication, the nurse should anticipate what effect? A) Constricted pupils B) Dilated bronchioles C) Decreased peristaltic movement D) Relaxed muscular walls of the urinary bladder

Ans: A Feedback: Parasympathetic stimulation results in constricted pupils, constricted bronchioles, increased peristaltic movement, and contracted muscular walls of the urinary bladder.

A 70-year-old patient in the ICU has become agitated and inattentive since his heart surgery. The nurse knows that this ICU psychosis frequently occurs in individuals with pre-existing dementia, history of alcohol abuse, and severe disease. What interventions should the nurse provide this patient to improve the patient's cognition (select all that apply)? A. Improve oxygenation. B. Provide a small amount of beer. C. Have the family stay with the patient. D. Enable the patient to sleep on a schedule with dim lights. E. Decrease sensory overload by conversing away from patient's room.

Ans. A, D, E ICU psychosis is from delirium in most ICU patients. Improving oxygenation, enabling the patient to sleep, and decreasing sensory overload along with orientation is all helpful in improving the patient's cognition. The beer may or may not be allowed for this patient, and the nurse should not assume that it will help. Having a family member stay with the patient to reorient the patient is helpful, but the family group may increase sensory overload with conversations not involving the patient.

A gerontologic nurse educator is providing practice guidelines to unlicensed care providers. Because reaction to painful stimuli is sometimes blunted in older adults, what must be used with caution? A) Hot or cold packs B) Analgesics C) Anti-inflammatory medications D) Whirlpool baths

Ans: A Feedback: Reaction to painful stimuli may be decreased with age. Because pain is an important warning signal, caution must be used when hot or cold packs are used. The older patient may be burned or suffer frostbite before being aware of any discomfort. Any medication is used with caution in the elderly, but not because of the decreased sense of heat or cold. Whirlpool baths are generally not a routine treatment ordered for the elderly.

A patient exhibiting an uncoordinated gait has presented at the clinic. Which of the following is the most plausible cause of this patient's health problem? A) Cerebellar dysfunction B) A lesion in the pons C) Dysfunction of the medulla D) A hemorrhage in the midbrain

Ans: A Feedback: The cerebellum controls fine movement, balance, position sense, and integration of sensory input. Portions of the pons control the heart, respiration, and blood pressure. Cranial nerves IX through XII connect to the brain in the medulla. Cranial nerves III and IV originate in the midbrain.

A patient is admitted to the medical unit with an exacerbation of multiple sclerosis. When assessing this patient, the nurse has the patient stick out her tongue and move it back and forth. What is the nurse assessing? A) Function of the hypoglossal nerve B) Function of the vagus nerve C) Function of the spinal nerve D) Function of the trochlear nerve

Ans: A Feedback: The hypoglossal nerve is the 12th cranial nerve. It is responsible for movement of the tongue. None of the other listed nerves affects motor function in the tongue.

The patient in the ED has just had a diagnostic lumbar puncture. To reduce the incidence of a post-lumbar puncture headache, what is the nurse's most appropriate action? A) Position the patient prone. B) Position the patient supine with the head of bed flat. C) Position the patient left side-lying. D) Administer acetaminophen as ordered.

Ans: A The lumbar puncture headache may be avoided if a small-gauge needle is used and if the patient remains prone after the procedure. Acetaminophen is not administered as a preventative measure for post-lumbar puncture headaches.

A nurse is caring for a patient diagnosed with Ménière's disease. While completing a neurologic examination on the patient, the nurse assesses cranial nerve VIII. The nurse would be correct in identifying the function of this nerve as what? A) Movement of the tongue B) Visual acuity C) Sense of smell D) Hearing and equilibrium

Ans: D Feedback: Cranial nerve VIII (acoustic) is responsible for hearing and equilibrium. Cranial nerve XII (hypoglossal) is responsible for movement of the tongue. Cranial nerve II (optic) is responsible for visual acuity and visual fields. Cranial nerve I (olfactory) functions in sense of smell.

A patient is brought to the ER following a motor vehicle accident in which he sustained head trauma. Preliminary assessment reveals a vision deficit in the patient's left eye. The nurse should associate this abnormal finding with trauma to which of the following cerebral lobes? A)Temporal B)Occipital C)Parietal D)Frontal

B The posterior lobe of the cerebral hemisphere is responsible for visual interpretation. The temporal lobe contains the auditory receptive areas. The parietal lobe contains the primary sensory cortex, and is essential to an individual's awareness of the body in space, as well as orientation in space and spatial relations. The frontal lobe functions in concentration, abstract thought, information storage or memory, and motor function.

A patient scheduled for magnetic resonance imaging (MRI) has arrived at the radiology department. The nurse who prepares the patient for the MRI should prioritize which of the following actions? A) Withholding stimulants 24 to 48 hours prior to exam B) Removing all metal-containing objects C) Instructing the patient to void prior to the MRI D) Initiating an IV line for administration of contrast

B Patient preparation for an MRI consists of removing all metal-containing objects prior to the examination. Withholding stimulants would not affect an MRI; this relates to an electroencephalography (EEG). Instructing the patient to void is patient preparation for a lumbar puncture. Initiating an IV line for administration of contrast would be done if the patient was having a CT scan with contrast.

A gerontologic nurse planning the neurologic assessment of an older adult is considering normal, age-related changes. Of what phenomenon should the nurse be aware? A) Hyperactive deep tendon reflexes B) Reduction in cerebral blood flow C) Increased cerebral metabolism D) Hypersensitivity to painful stimuli

B Reduction in cerebral blood flow (CBF) is a change that occurs in the normal aging process. Deep tendon reflexes can be decreased or, in some cases, absent. Cerebral metabolism decreases as the patient advances in age. Reaction to painful stimuli may be decreased with age. Because pain is an important warning signal, caution must be used when hot or cold packs are used.

The nurse has admitted a new patient to the unit. One of the patient's admitting orders is for an adrenergic medication. The nurse knows that this medication will have what effect on the circulatory system? A) Thin, watery saliva B) Increased heart rate C) Decreased BP D) Constricted bronchioles

B The term "adrenergic" refers to the sympathetic nervous system. Sympathetic effects include an increased rate and force of the heartbeat. Cholinergic effects, which correspond to the parasympathetic division of the autonomic nervous system, include thin, watery saliva, decreased rate and force of heartbeat, and decreased BP.

4 Pulse oximetry is a noninvasive and continuous method of determining the oxygen saturation of SpO2. Monitoring SpO2 may reduce the frequency of arterial blood gas (ABG) sampling. SpO2 is normally 95% to 100%. A value less than that may indicate hypoperfusion. Text Reference - p. 1609

The health care provider requests constant hemoglobin (SpO2) monitoring for a patient. What is the normal range of SpO2, which indicates that the saturation pressure of oxygen in this patient is adequate? 1 80-85% 2 85-90% 3 90-95% 4 95-100%

1 Critical care nurses will face ethical dilemmas related to the care of patients. This can occur over perceived issues of delivering futile or nonbeneficial care, such as attempting to resuscitate a neonate born at 21 weeks of gestation. This situation does not represent a societal, temporal, or sequential dilemma. Text Reference - p. 1599

The nurse in the newborn intensive care unit (NICU) is providing care to a neonate born at 21 weeks gestation who is being resuscitated in the delivery room. The standard gestation compatible with life is 23 to 24 weeks gestation. Which type of dilemma could the nurse be facing? 1 Ethical 2 Societal 3 Temporal 4 Sequential

2 Inadequate pain control is often linked with agitation and anxiety and is known to add to the stress response. Pain is associated with infection, but the patient is at the highest risk for developing anxiety. Dementia and depression are not associated with unrelieved pain. Text Reference - p. 1600

The nurse is assessing a patient in the intensive care unit (ICU). The patient is intubated and exhibiting symptoms of pain. Based on this data what condition is the patient at the highest risk for developing? 1 Anxiety 2 Infection 3 Dementia 4 Depression

The nurse is caring for a patient intubated and on a mechanical ventilator for several days. Which weaning parameter would tell the nurse if the patient has enough muscle strength to breathe without assistance? Tidal volume Minute ventilation Forced vital capacity Negative inspiratory force

Negative inspiratory force The negative inspiratory force measures inspiratory muscle strength. Tidal volume and minute ventilation assess the patient's respiratory endurance. Forced vital capacity is not used as a measure to determine weaning from a ventilator

The nurse in collaboration with respiratory therapy is determining a patient's readiness to wean from the ventilator. Which finding indicates the patient is not a candidate for weaning (select all that apply.)? Minute volume of 8 L/min Patient follow commands Serum hemoglobin of 6 g/dL Respirations of 28 breaths/min Mean arterial pressure (MAP) of 45 mm Hg Negative inspiratory force (NIF) of -15 cm H2O

Serum hemoglobin of 6 g/dL Mean arterial pressure (MAP) of 45 mm Hg Negative inspiratory force (NIF) of -15 cm H2O Findings that support readiness for weaning are minute volume of 8 L/min, patient is alert and follow commands, and respirations of 28 breaths/min. Findings that indicate the patient is not ready for weaning include serum hemoglobin of 6 g/dL, mean arterial pressure (MAP) of 45 mm Hg, and negative inspiratory force (NIF) of -15 cm H2O. Extubating a patient with severe anemia, poor perfusion, and weakened breathing effort will likely result in poor outcomes such as worsening of condition and reintubation.

2 Positive end-expiratory pressure (PEEP) therapy is given to patients with pulmonary edema to provide a counter pressure opposing fluid extravasation. PEEP is not used for patients with hypovolemia, low cardiac output and unilateral or nonuniform lung disease because in those patients, the adverse effects of PEEP may outweigh any benefits. Text Reference - p. 1621

What condition would the nurse infer that a patient undergoing positive end-expiratory pressure (PEEP) therapy has? 1 Hypovolemia 2 Pulmonary edema 3 Low cardiac output 4 Unilateral lung disease

2 Arterial pressure-based cardiac output (APCO) monitoring uses the arterial waveform characteristics along with patient demographic data including gender, age, height, and weight to calculate stroke volume (SV). Basal metabolic rate (BMR), body mass index (BMI), blood pressure, and heart rate are not used to calculate stroke volume with this measuring device. Heart rate is used to calculate continuous cardiac output (COO) and continuous cardiac index (CCI). Text Reference - p. 1607

What data are used to calculate stroke volume (SV) for a patient with arterial pressure-based cardiac output (APCO) monitoring? 1 Height, BMR, age, gender 2 Gender, age, height, weight 3 BMI, BMR, blood pressure, heart rate 4 Age, gender, blood pressure, heart rate

37.5 The formula to calculate the rapid shallow breathing index is f/VT. Therefore, rapid shallow breathing index = 15/0.4 = 37.5 breaths/min/L. Text Reference - p. 1626

What is the rapid shallow breathing index of a patient whose spontaneous respiratory rate (f) is 15 breaths/min and spontaneous tidal volume (VT) is 0.4 L? Record your answer using one decimal place. ___________________breaths/min/L

3, 4 Suctioning is preceded by a thorough assessment and hyperoxygenation for 30 seconds. Sterile, not clean, gloves are necessary and it is not necessary to administer a bronchodilator. Instillation of normal saline into the ET tube is not an accepted standard practice. Text Reference - p. 1616

Which interventions should the nurse perform before suctioning a patient who has an endotracheal (ET) tube using open-suction technique? Select all that apply. 1 Put on clean gloves. 2 Administer a bronchodilator. 3 Perform a cardiopulmonary assessment. 4 Hyperoxygenate the patient for 30 seconds. 5 Insert a few drops of normal saline into the ET to break up secretions.

1 Patients with preexisting dementia, such as Alzheimer's disease, are at an increased risk for developing delirium when receiving care in the intensive care unit (ICU). Diabetes mellitus, Parkinson's disease, and multiple sclerosis are not known risk factors for developing delirium. Text Reference - p. 1601

Which is the recommendation from the American Association of Critical Care Nursing (AACN) regarding family visitation in the intensive care unit (ICU)? 1 Individualized visitation 2 Visitation on the evening shift 3 Visitation as prescribed by the physician 4 Hourly visitations occurring on each shift

1 In a patient with a head injury, positive pressure ventilation decreases the venous return because of the increase in intrathoracic pressure. Increased intrathoracic pressure causes jugular vein distension rather than compression. Positive pressure ventilation increases the cerebral volume. A decrease in venous return causes an increase in intracranial pressure. Text Reference - p. 1623

Which neurologic complication may occur in a patient with a head injury who is on positive pressure ventilation? 1 Decrease in venous return 2 Compression of jugular vein 3 Reduction of cerebral volume 4 Reduction of intracranial pressure

Ventricular assist devices are designed to a. provide permanent, total circulatory support when the left ventricle fails. b. temporarily partially or totally support circulation until a donor heart can be obtained. c. support circulation only when patients cannot be weaned from cardiopulmonary bypass. d. reverse the effects of circulatory failure in patients with acute myocardial infarction (MI) in cardiogenic shock.

b. Rationale: Ventricular assist devices are temporary devices that can partially or totally support circulation until the heart recovers and can be weaned from cardiopulmonary bypass or when a donor heart can be obtained. The devices currently available do not permanently support circulation.

The nurse suctions the patient's ET tube when the patient a. has peripheral crackles in all lobes. b. has not been suctioned for 2 hours. c. has coarse rhonchi over central airways. d. needs stimulation to cough and deep-breathe.

c. Rationale: Suctioning an ET tube is performed when adventitious sounds over the trachea or bronchi confirm the presence of secretions that can be removed by suctioning. Visible secretions in the ET tube, respiratory distress, suspected aspiration, increase in peak airway pressures, and changes in oxygen status are other indications. Peripheral crackles are not an indication for suctioning, and suctioning as a means of inducing a cough is not recommended because of the complications associated with suctioning.

The nurse responds to a ventilator alarm and finds the patient lying in bed gasping and holding the endotracheal tube (ET) in her hand. Which action should the nurse take next? a. Activate the rapid response team. b. Provide reassurance to the patient. c. Call the health care provider to reinsert the tube. d. Manually ventilate the patient with 100% oxygen.

d. Manually ventilate the patient with 100% oxygen. The nurse should ensure maximal patient oxygenation by manually ventilating with a bag-valve-mask system. Offering reassurance to the patient, notifying the health care provider about the need to reinsert the tube, and activating the rapid response team are also appropriate after the nurse has stabilized the patient's oxygenation.

During hemodynamic monitoring, the nurse finds that the patient has a decreased CO with unchanged pulmonary artery wedge pressure (PAWP), HR, and SVR. The nurse identifies that the patient has a decrease in a. SV. b. preload. c. afterload. d. contractility.

d. Rationale: Cardiac output (CO) is dependent on heart rate and stroke volume, and stroke volume is determined by preload, afterload, and contractility. If CO is decreased and heart rate is unchanged, stroke volume is the variable factor. If the preload determined by pulmonary artery wedge pressure (PAWP) and the afterload determined by SVR are unchanged, the factor that is changed is the contractility of the myocardium.

Four hours after mechanical ventilation is initiated, a patient's arterial blood gas (ABG) results include a pH of 7.51, PaO2 of 82 mm Hg, PaCO2 of 26 mm Hg, and HCO3- of 23 mEq/L (23 mmol/L). The nurse will anticipate the need to a. increase the FIO2. c. increase the respiratory rate. b. increase the tidal volume. d. decrease the respiratory rate.

d. decrease the respiratory rate. The patient's PaCO2 and pH indicate respiratory alkalosis caused by too high a respiratory rate. The PaO2 is appropriate for a patient with COPD and increasing the respiratory rate and tidal volume would further lower the PaCO2.

A patient is receiving mechanical ventilation after having a stroke. The nurse determines that the ventilator settings are based on which patient status? 1 Ideal body weight, vital signs, and family preference 2 Ethics committee results, current physiologic state, and ideal body weight 3 Respiratory muscle strength, ethics committee results, and family preference 4 Arterial blood gases (ABGs), current physiologic state, and respiratory muscle strength

4 Arterial blood gases (ABGs), current physiologic state, and respiratory muscle strength Settings on mechanical ventilators are based on the patient's physiologic status, such as ABGs, ideal body weight, current physiologic state, level of consciousness, and respiratory strength. Ethics committee results and family preference are psychosocial in nature and are not criteria used to determine mechanical ventilation settings.

A patient is diagnosed with an exacerbation of chronic pulmonary disease. What is an appropriate nursing action? 1 Consult with the agency's ethics committee. 2 Warn the family about the patient's need for ventilator support. 3 Discuss how critically ill the patient will become without ventilator support. 4 Discuss mechanical ventilation with the patient, family, and healthcare providers.

4 Discuss mechanical ventilation with the patient, family, and healthcare providers. The nurse should encourage all patients with chronic illnesses to discuss the possibility of mechanical ventilation with their families and health care providers. The patient may or may not become critically ill without ventilator support. The decision to use, withhold, or withdraw mechanical ventilation will be made carefully, respecting the wishes of the patient and caregiver; however, if disagreements occur, the agency's ethics committee may be consulted for assistance.

The nurse is teaching the patient's caregiver about receiving positive pressure ventilation. What movements should the nurse tell the caregiver to avoid doing to the patient? 1 Arm circles 2 Knee bends 3 Quadriceps setting 4 External rotation of the hip

4 External rotation of the hip The nurse should advise the caregiver to avoid external rotation of the patient's hip; this movement can be avoided by properly positioning the patient and by the use of specialized mattresses and beds. Simple maneuvers such as arm circles, knee bends and quadriceps setting should be performed, because they maintain the muscle tone in the upper and lower extremities of the patient.

When taking care of a patient diagnosed with respiratory failure on a mechanical ventilator, the nurse hears the apnea alarm beeping. What assessment data should be gathered to determine the cause of the alarm? 1 Pain or anxiety 2 Partial ventilator disconnect 3 Secretions, coughing, or gagging 4 Oversedation with opioid analgesics

4 Oversedation with opioid analgesics The apnea alarm on mechanical ventilation may be caused by respiratory arrest, oversedation, change in patient condition, or loss of airway (total or partial extubation). The high-pressure limit alarm is caused by secretions, coughing, or gagging. The low tidal volume alarm can be caused by partial ventilator disconnect. The high tidal volume alarm can be caused by pain or anxiety.

A patient in the intensive care unit has been intubated for the relief of airway obstruction. What nursing actions should be performed to prevent complications after intubation? Select all that apply. 1 Obtain a chest x-ray exam to confirm the placement. 2 Immediately catheterize the patient and check for urine output. 3 Obtain a computed tomography (CT) scan to note the placement. 4 Auscultate lungs bilaterally and also epigastrium for breath sounds. 5 Use an end-tidal carbon dioxide detector to note presence of exhaled carbon dioxide.

1 Obtain a chest x-ray exam to confirm the placement. 4 Auscultate lungs bilaterally and also epigastrium for breath sounds. 5 Use an end-tidal carbon dioxide detector to note presence of exhaled carbon dioxide. Following an intubation, it is important to confirm the placement of the endotracheal (ET) tube. This confirmation is obtained by x-ray after visualizing the ET tube correctly placed in the trachea. Auscultating lungs for breath sounds confirms that air is going into the lungs and not in the stomach. If the sounds are heard over the epigastrium, it indicates that the ET tube has gone in the stomach. Presence of carbon dioxide in exhaled air also confirms that the tube has gone into the lungs, and the breathing effort is normal. In this case, a CT scan is redundant. However, an x-ray is sufficient to confirm the placement of the ET tube. The patient may require a urinary catheter, but it is not an immediate intervention and can be done after intubation.

patient is weaned from artificial ventilation. What interventions should the nurse perform during weaning? Select all that apply. 1 Obtain baseline vital signs and respiratory parameters. 2 Make the patient walk a bit and then proceed with the trial. 3 Anesthetize or restrain the patient to avoid any resistance. 4 Place the patient in a comfortable sitting or semirecumbent position. 5 Closely monitor for signs and symptoms that may signal intolerance and a need to end the trial.

1 Obtain baseline vital signs and respiratory parameters. 4 Place the patient in a comfortable sitting or semirecumbent position. 5 Closely monitor for signs and symptoms that may signal intolerance and a need to end the trial. During a weaning trial, the patient is placed in a comfortable sitting or semirecumbent position. Baseline vital signs and respiratory parameters should be obtained. The patient should be closely monitored for signs and symptoms that may signal intolerance and a need to end the trial. The patient should not be anesthetized or made to walk

An endotracheal (ET) tube is inserted in a patient. The nurse inflates the cuff to stabilize the tube. How much cuff pressure should be maintained to keep it inflated and ensure adequate tracheal perfusion? 1 10-15 cm H2O 2 20-25 cm H2O 3 30-35 cm H2O 4 40-45 cm H2O

2 20-25 cm H2O To ensure adequate tracheal perfusion, the nurse should maintain cuff pressure at 20 to 25 cm H2O. Excess cuff pressure can damage the tracheal mucosa. Lesser cuff pressure may cause the ET tube to become destabilized and extubate.

An older adult patient reports having used an "iron lung" after contracting polio as a child. The nurse knows this patient is referring to which type of mechanical ventilation? 1 Positive pressure 2 Negative pressure 3 Volume ventilation 4 Pressure ventilation

2 Negative pressure The "iron lung" was the first form of negative pressure ventilation, developed during the polio epidemic. Negative pressure uses the chambers encasing the chest and surrounding it with intermittent negative pressure; expiration is passive, and the negative pressure is delivered by noninvasive measures. Pressure ventilation means the peak inspiratory pressure is predetermined and the tidal volume delivered varies based on the patient. Volume ventilation has a predetermined tidal volume delivered with each inspiration and the pressure varies based on the patient. Positive pressure ventilation is the primary method used with acutely ill patients, where air is pushed into the lungs during inspiration under positive pressure.

The nurse is caring for a group of patient's in the intensive care unit. Which patient is a candidate for bilevel positive airway pressure (BiPAP)? 1 Patient with shock 2 Patient with sleep apnea 3 Patient with altered mental status 4 Patient with increased airway secretions

2 Patient with sleep apnea Bilevel positive airway pressure (BiPAP) is used for patients with sleep apnea. Patients with shock, altered mental status and/or increased airway secretions are not candidates for BiPAP because of the risk of aspiration and the inability to remove the mask.

The nurse is assessing a patient placed on mechanical ventilation and hears breath sounds on the right but not on the left side of the chest. What common complication should the nurse immediately notify the health care provider about? 1 Hypertension 2 Pneumothorax 3 Electrolyte imbalance 4 Increased cardiac output

2 Pneumothorax Mechanical ventilation can cause pneumothorax as a result of excessive pressure applied to lung tissue. Hypertension is not a direct complication; however, a patient undergoing mechanical ventilation may be anxious and fearful, resulting in high blood pressure; sedation should be considered in this event. Electrolyte imbalance is not a related complication. Mechanical ventilation does increase intrathoracic pressure, which may then increase cardiac output, causing a beneficial secondary effect.

While evaluating a mechanically ventilated patient, the nurse notes that the auto-PEEP has been activated. Which mode of mechanical ventilation does the nurse suspect? 1 Volume mode 2 Pressure mode 3 Continuous positive airway pressure 4 Positive end-expiratory pressure mode

2 Pressure mode The auto-PEEP is a pressure mode that provides a pressure-limited breath delivered at a set rate that may permit spontaneous breathing. The positive end-expiratory pressure (PEEP) mode is a ventilator maneuver in which positive pressure is applied to the airway during exhalation. Continuous positive airway pressure (CPAP) is similar to PEEP, but the pressure is delivered continuously during spontaneous breathing, preventing the patient's airway pressure from falling to zero. Volume modes require that rate, tidal volume, inspiratory time, sensitivity, and/or PEEP are set for the patient

The nurse working in the intensive care unit (ICU) is taking care of a patient on a mechanical ventilator who had a motor vehicle accident two weeks ago. What does the nurse know about this situation? 1 The patient has severe hypoxia due to acute respiratory failure. 2 The ventilator will support the patient until he or she can breathe on his or her own. 3 The patient suffered from a chronic pulmonary disease before the accident. 4 The patient will be on long-term ventilation until the family decides to withdraw ventilator support.

2 The ventilator will support the patient until he or she can breathe on his or her own. Mechanical ventilation is not curative. It is a means of supporting patients until they recover the ability to breathe independently. The decision to use, withhold, or withdraw mechanical ventilation will be made carefully, involving the agency's ethics committee for assistance. The patient's medical history or diagnosis is not known; the nurse does not know if the patient suffered from chronic pulmonary disease or if he has severe hypoxia.

The nurse working in a critical care unit understands that tidal volume is an important setting in a mechanical ventilator. Which statement appropriately describes tidal volume? 1 Number of breaths the ventilator delivers per minute 2 Volume of gas delivered to patient during each ventilator breath 3 Positive pressure used to augment patient's inspiratory pressure 4 Positive pressure applied at the end of expiration of ventilator breaths

2 Volume of gas delivered to patient during each ventilator breath Tidal volume is the volume of gas delivered to a patient during each ventilator breath. The number of breaths the ventilator delivers per minute is called the respiratory rate. The positive pressure used to augment the patient's inspiratory pressure is called pressure support. The positive pressure applied at the end of expiration of ventilator breaths is called positive end-expiratory pressure.

When planning care for a patient on a mechanical ventilator, the nurse understands that the application of positive end-expiratory pressure (PEEP) to the ventilator settings has which therapeutic effect? 1 Increased inflation of the lungs 2 Prevention of barotrauma to the lung tissue 3 Prevention of alveolar collapse during expiration 4 Increased fraction of inspired oxygen concentration (FIO2) administration

3 Prevention of alveolar collapse during expiration PEEP is positive pressure that is applied to the airway during exhalation. This positive pressure prevents the alveoli from collapsing, improving oxygenation and enabling a reduced FIO2 requirement. PEEP does not cause increased inflation of the lungs or prevent barotrauma. Auto-PEEP resulting from inadequate exhalation time may contribute to barotrauma.

When assessing the settings of a patient's ventilator, the nurse knows that which parameter is abnormal? 1 PaO2 of 66 mm Hg 2 PEEP of 5 cm H 2O 3 Tidal volume of 12 mL/kg 4 Respiratory rate of 20 breaths/minute

3 Tidal volume of 12 mL/kg Usual tidal volume is 6-10 mL/kg; a tidal volume of 12 ml/kg is abnormally high. A respiratory rate of 20 breaths/minute is within normal; usual settings are 6-20 breaths/minute. A PaO2 level of 66 mm Hg is normal; the usual is greater than 60 mm Hg. PEEP of 5 cm H2O is the usual setting.

1 Prone positioning refers to the repositioning of a patient from a supine or lateral position to a prone position. This repositioning improves lung reexpansion through various mechanisms. Firstly, the gravity reverses the effects of fluid in the dependent parts of the lungs as the patient is moved from supine to prone. Secondly, in the prone position, the heart rests on the sternum, away from the lungs, contributing to an overall uniformity of pleural pressures. These two mechanisms help in better ventilation in the patient with respiratory failure. The prone position is a relatively safe supportive therapy used for critically ill patients with acute lung injury or ARDS and is used for improved oxygenation. Resting in other positions such as sitting, supine, or lateral may not help in oxygenation. Text Reference - p. 1622

A nurse is attending a patient with acute respiratory distress syndrome (ARDS). Which position is best for this patient? 1 Prone 2 Sitting 3 Supine 4 Lateral

1, 2, 5 It is extremely essential to monitor all ICU patients and prevent delirium. Sensory overload can lead to patient distress and anxiety. The nurse should limit noise in the ICU and help the patient to understand that some noises in the ICU cannot be prevented, for example, beeping of a cardiac monitor. The nurse can also limit noise levels by muting phones, setting alarms based on the patient's condition, and reducing unnecessary alarms. The use of clocks and calendars can help orient the patient to time and date. Regular assessment should be carried out using tools like the Confusion Assessment Method for the ICU and the Intensive Care Delirium Screening Checklist. Seeing a familiar face may make the patient comfortable; therefore, the presence of a caregiver is important. Giving regular sponge baths helps to maintain hygiene but doesn't affect delirium directly. Text Reference - p. 1601

A nurse is caring for a patient in ICU who is taking sedatives. What are the steps that a nurse should take in order to prevent delirium in this patient? Select all that apply. 1 Keep the noise in the ICU to a minimum. 2 Use clocks and a calendar to keep the patient oriented. 3 Ensure that there is minimal communication with the patient. 4 Give regular sponge baths to the patient, and monitor the urinary output. 5 Carry out frequent assessment for delirium by using the Confusion Assessment Method.

1, 2, 3 The primary goal of nutritional support is to prevent or correct nutritional deficiencies. This is usually done by the early provision of enteral nutrition or parenteral nutrition. Enteral nutrition preserves the structure and function of the gut mucosa and stops the movement of gut bacteria across the intestinal wall and into the bloodstream. In addition to this, early enteral nutrition is associated with fewer complications. Enteral feedings cannot be administered to all patients; in patients with paralytic ileus, intestinal obstruction, and GI ischemia, enteral feeding is contraindicated. In these patients, parenteral feeding is the best option. Text Reference - p. 1600

A nurse is starting enteral feeding through a nasogastric tube for a patient in ICU. What advantages of enteral feeding over parenteral feeding does the nurse identify in the patient? Select all that apply. 1 Preserves the structure and function of gut mucosa 2 Stops the movement of gut bacteria across the intestinal wall 3 Results in fewer complications 4 Prevents and corrects nutritional deficiencies 5 Can be administered to all patients

1, 3, 4 It is extremely important for a nurse to closely assess the patient before, during, and after the suctioning procedure. If the patient is unable to tolerate suctioning, stop the procedure and hyperoxygenate until equilibration occurs before attempting next suction pass. Decreased SpO2, increased or decreased BP, and development of dysrhythmias are indicators that the patient is not tolerating suction. Sustained coughing rather than absence of coughing also indicates that the patient is not tolerating suctioning. Presence of shivering and convulsions is not related to suctioning. Text Reference - p. 1616

A nurse is suctioning a patient. Which signs indicate that the patient is not tolerating suctioning? Select all that apply. 1 Decreased SpO2 2 Absence of coughing 3 Increased blood pressure (BP) 4 Development of dysrhythmias 5 Shivering and convulsions of the entire body

4 Central venous pressure (CVP) is a measure of the filling pressure of the right ventricle and is indicative of how the right side of the heart accommodates fluid load. A series of CVP measurements of 12 mm Hg or higher indicates failure of the right ventricle to handle venous return. A normal CVP measurement is 2 to 8 mm Hg. Cardiogenic shock and circulatory failure are late manifestations of heart failure in general and would likely show a decreased CVP and cardiac output. CVP may be increased with left ventricular failure; however, this is a late sign. It is possible to have both right and left failure at the same time. Text Reference - p. 1608

A nurse measures a patient's central venous pressure and recognizes a series of increased readings as directly indicative of: 1 Cardiogenic shock 2 Circulatory failure 3 Left ventricular failure 4 Right ventricular failure

2 Mechanical ventilation can cause pneumothorax as a result of excessive pressure applied to lung tissue. Hypertension is not a direct complication; however, a patient undergoing mechanical ventilation may be anxious and fearful, resulting in high blood pressure; sedation should be considered in this event. Electrolyte imbalance is not a related complication. Mechanical ventilation does increase intrathoracic pressure, which may then increase cardiac output, causing a beneficial secondary effect. Text Reference - p. 1615

A nurse should monitor a patient undergoing mechanical ventilation for which common complication? 1 Hypertension 2 Pneumothorax 3 Electrolyte imbalance 4 Increased cardiac output

1, 4, 5 It is important to remove the dentures of the patient during the process of oral intubation because the dentures can obstruct the airway. Inform the patient that brief restraint will be necessary for safety purposes. It is also necessary to brief the patient about the procedure to avoid any type of resistance during intubation. Metallic objects on the body do not interfere with the procedure of intubation, and, therefore, need not be removed. Before intubating, it is extremely important to preoxygenate the patient with 100% oxygen. This is because the patient will not get any oxygen supply during intubation for a short period. STUDY TIP: Avoid planning other activities that will add stress to your life between now and the time you take the licensure examination. Enough will happen spontaneously; do not plan to add to it. Text Reference - p. 1614

A patient admitted to the ICU is being intubated. What are the steps that a nurse should ensure for a safe intubation? Select all that apply. 1 Remove the dentures of the patient. 2 Ensure that the patient is not wearing any metallic objects. 3 Oxygenate the patient using a bag-valve-mask (BVM) and 95% oxygen before the procedure. 4 Inform the patient that brief restraint will be necessary. 5 Explain the procedure to the patient and also the patient's role in the procedure.

2, 3, 5 It is extremely important to follow aseptic measures to avoid infection following an IABP. Covering all the insertion sites with occlusive dressings avoids infections. Aseptic techniques should be followed during insertion and dressing changes to prevent infection. Prophylactic antibiotics prevent infections. Infection doesn't breed in the dressings if aseptic precautions are followed and the dressings are cleaned and replaced regularly. Replacing the lines every two hours is not necessary, but following aseptic conditions is. A culture swab helps to identify the presence of infection but d

A patient has received intraaortic balloon pump (IABP) therapy. In this case, what precautions should a nurse take to prevent any infection at the site? Select all that apply. 1 Replace the lines every two to three hours. 2 Cover all insertion sites with occlusive dressings. 3 Use strict aseptic technique line insertion and dressing changes. 4 Send culture swabs from the insertion site regularly. 5 Administer prophylactic antibiotics for the entire course of therapy

1 The distal lumen port (catheter tip), labeled A in the image, is within the pulmonary artery. This port is used to monitor pulmonary artery (PA) pressures. Choice B is the port used for infusions. Choice C is the port used for injecting medications. Choice D is the port used to inflate the balloon. Text Reference - p. 1607

A patient has the following device. Which port should be used to measure pulmonary artery pressure? 1. A 2. B 3. C 4. D

1, 4, 5 Following an intubation, it is important to confirm the placement of the endotracheal (ET) tube. This confirmation is obtained by x-ray after visualizing the ET tube correctly placed in the trachea. Auscultating lungs for breath sounds confirms that air is going into the lungs and not in the stomach. If the sounds are heard over the epigastrium, it indicates that the ET tube has gone in the stomach. Presence of carbon dioxide in exhaled air also confirms that the tube has gone into the lungs, and the breathing effort is normal. In this case, a CT scan is redundant. However, an x-ray is sufficient to confirm the placement of the ET tube. The patient may require a urinary catheter, but it is not an immediate intervention and can be done after intubation. Text Reference - p. 1614

A patient in ICU has been intubated for the relief of airway obstruction. What nursing actions should be performed to prevent complications after intubation? Select all that apply. 1 Obtain a chest x-ray to confirm the placement. 2 Obtain a computed tomography (CT) scan to note the placement. 3 Immediately catheterize the patient and check for urine output. 4 Auscultate lungs bilaterally and also epigastrium for breath sounds. 5 Use an end-tidal carbo

3 PaCO2 is the best indicator of alveolar hyperventilation or hypoventilation. Continuous PETCO2 monitoring can assess the patency of the airway and the presence of breathing. Continuous oxygen saturation (SpO2) provides objective data regarding tissue oxygenation. Central venous pressure (CVP) or pulmonary artery (PA) catheters with ScvO2 or SvO2 capability provide an indirect indication of the patient's tissue oxygenation status. Text Reference - p. 1615

A patient is admitted to the ICU and is on assisted ventilation. Which is the best indicator of inadequate alveolar oxygenation? 1 PETCO2 2 SpO2 3 PaCO2 4 ScvO2 or SvO2

1, 3, 4 Before PA catheter insertion, the patient is positioned supine and flat. The procedure is explained to the patient, and informed consent is obtained. The patient's electrolyte, acid-base, oxygenation, and coagulation status are noted. Imbalances such as hypokalemia, hypomagnesemia, hypoxemia, or acidosis can make the heart more irritable and increase the risk of ventricular dysrhythmia during catheter insertion. Coagulopathy increases the risk of hemorrhage. The procedure is never performed in a sitting position. The PA catheter is inserted through a sheath percutaneously into the internal jugular, subclavian, antecubital, or femoral vein using surgical asepsis. The insertion sites have to be dressed with occlusive dressings. Text Reference - p. 1608

A patient is advised to have a pulmonary artery (PA) catheter inserted for pulmonary artery pressure monitoring. What precautions are necessary prior to insertion of the catheter? Select all that apply. 1 Place the patient in the supine and flat position. 2 Position the patient sitting and with head turned laterally. 3 Explain the procedure to the patient and get consent. 4 Note the patient's electrolyte levels and oxygenation and coagulation status. 5 Do not cover the catheter insertion site with any dressings.

2, 4, 5 Appropriate patient selection for ventricular assist device (VAD) includes patients who are waiting for heart transplantation, who are diagnosed with Class IV heart disease, and have failed medical therapy, and who have failed to wean from cardiopulmonary bypass (CPB). Body surface area less than 1.3 m2 is a contraindication for ventricular assist device (VAD) therapy. Liver failure unrelated to a cardiac event is a contraindication for ventricular assist device (VAD) therapy. Text Reference - p. 1612

A patient is being considered for ventricular assist device (VAD) therapy. Which criteria indicate that this patient is an appropriate candidate for implantation of this device? Select all that apply. 1 Body surface area 1.1 m2 2 Placed on the heart transplantation list 3 Diagnosed with alcoholic liver failure 4 Diagnosed with Class IV heart disease 5 Unable to wean from the cardiopulmonary bypass (CPB) machine

3 Presence of condensate or water in tubing triggers a high-pressure ventilation alarm. Power failure triggers ventilator inoperative or low battery alarm. Insufficient gas flow and tracheotomy cuff leak triggers low tidal volume or minute ventilation alarm. Text Reference - p. 1620

A patient is being mechanically ventilated. A high-pressure ventilation alarm sounds. The nurse should assess for what cause of this type of alarm? 1 Power failure 2 Insufficient gas flow 3 Condensate in tubing 4 Tracheotomy cuff leak

3 Before inserting a line into the radial artery, an Allen test should be performed to confirm that ulnar circulation to the hand is adequate. In this test, pressure is applied to the radial and ulnar arteries simultaneously. The patient opens and closes the hand repeatedly until the hand blanches. When the pressure on the ulnar artery is released, the hand should return to a pink color within six seconds. If pinkness does not return within six seconds the ulnar artery is inadequate to maintain blood flow to the extremity and the radial artery should not be used for arterial line insertion. The phlebostatic axis is used to zero the arterial line, which would be done much later. Because of the risk of heparin-induced thrombocytopenia (HIT), heparinized saline should not be routinely used for the flush solution. The flush bag should be set to deliver 3 to 6 mL/hr. Text Reference - p. 1606

A patient is being prepared for insertion of an arterial measuring device. What should be done before the catheter is inserted into the patient's radial artery? 1 Locate the phlebostatic axis 2 Prepare a heparinized flush bag 3 Occlude the radial and ulnar arteries 4 Set an intravenous pump to deliver 15 mL/h

1, 3, 5 Nasal intubation is a blind procedure. There are chances that the tube may be misdirected. It may cause complications if there was a recent cranial surgery and fracture of the facial bones or the base of the skull. Suspected spinal fracture is a contraindication for oral intubation because it requires some movement of the neck and head. A deviated nasal septum can cause some difficulty in nasal intubation, although it is not a contraindication. STUDY TIP: Begin studying by setting goals. Make sure they are realistic. A goal of scoring 100% on all exams is not realistic, but scoring an 85% may be a better goal. Text Reference - p. 1614

A patient is being prepared for intubation using a nasal intubation technique. What absolute contraindications for nasal intubation should the nurse be aware of? Select all that apply. 1 Recent cranial surgery 2 Suspected spine injury 3 Fracture of facial bones 4 Deviated nasal septum 5 Fracture of the base of the skull

4 Indications for intraaortic balloon pump (IABP) therapy include acute myocardial infarction and cardiogenic shock. The use of the pump with this health problem allows time for emergent angiography. The pump is not used to reduce pressure in the pulmonary artery, improve coronary artery vessel perfusion, or to enhance the effectiveness of cardiac medications. Text Reference - p. 1610

A patient is experiencing cardiogenic shock after an acute myocardial infarction. Why would an intraaortic balloon pump (IABP) be beneficial for this patient? 1 Reduces pressure in the pulmonary artery 2 Improves coronary artery vessel perfusion 3 Enhances effectiveness of cardiac medications 4 Provides time for an emergency angiogram to be performed

2 Benzodiazepines, such as lorazepam, have anxioloytic activity and help to alleviate symptoms of anxiety in patients. Propofol is an anesthetic, and fentanyl is an analgesic agent. These two medications do not help to treat anxiety but are used to relax intubated, ventilated patients. Esomeprazole is a proton pump inhibitor that helps to reduce symptoms of peptic ulcer. Text Reference - p. 1600

A patient is experiencing symptoms of anxiety. The nurse anticipates that which medication will be prescribed? 1 Propofol 2 Lorazepam 3 Fentanyl 4 Esomeprazole

3 An SBT is recommended in patients who demonstrate weaning readiness. An SBT should be at least 30 minutes but no more than 120 minutes. At least 15 minutes but no more than 30 minutes, at least 30 minutes but no more than 60 minutes, and at least 60 minutes but no more than 120 minutes are not recommended time frames to determine weaning readiness. Text Reference - p. 1626

A patient is intubated. The nurse has to perform a spontaneous breathing trial (SBT) on this patient. For how long should this trial be done? 1 At least 15 minutes but not more than 30 minutes 2 At least 30 minutes but not more than 60 minutes 3 At least 30 minutes but not more than 120 minutes 4 At least 60 minutes but not more than 120 minutes

1, 2, 4 Patients have a higher risk for hospital-acquired pneumonia when they require mechanical ventilation. This is because the ET or tracheostomy tube bypasses the normal upper airway defenses. Additionally, poor nutritional state, immobility, and the underlying disease process make the patient more prone to infection. VAP is pneumonia that occurs 48 hours or more post-ET intubation. To prevent VAP, the health care team should strictly wash their hands before and after suctioning. An ET tube with a dorsal lumen above the cuff should be used to allow continuous suctioning of secretions in the subglottic area. Gloves should be worn whenever the nurse is in contact with the patient, and the nurse should change them frequently between activities to avoid cross-infection. If the ventilator circuit tubing is changed frequently, there is more risk of exposing the patient to various infections. Therefore, there should be no routine changes in the patient's ventilator circuit tubing. In addition to this, the head-of-bed should be elevated at a minimum of 30 to 45 degrees, unless medically contraindicated, to prevent pooling of secretions and facilitate suctioning. Text Reference - p. 1623

A patient is placed on a ventilator for assisted ventilation. What precautions should a nurse take to prevent the patient from ventilator-assisted pneumonia (VAP)? Select all that apply. 1 Wash hands before and after suctioning. 2 Use an endotracheal (ET) tube with a dorsal lumen above the cuff. 3 Change the patient's ventilator circuit tubing every two to three hours. 4 Wear gloves when in contact with the patient, and change gloves between activities. 5 Maintain the head-of-bed elevation at a minimum of 90 degrees unless medically contraindicated.

2, 4, 5, 6 The possible causes for a high-pressure limit alarm to go off include secretions, coughing, or gagging. It may also be set off in case of ventilator asynchrony if the patient is fighting the ventilator. Decreased compliance due to conditions like pulmonary edema may also cause the setting off of the high-pressure alarm. It can also be due to kinked or compressed tubing, which usually happens when the patient is biting on the endotracheal tube. Oversedation and loss of airway cause an apnea alarm and can also set off the low-pressure limit alarm. Text Reference - p. 1620

A patient is placed on mechanical ventilation. A nurse notices that the alarm for the high-pressure limit has been set off. What are the possible conditions that could give rise to this alarm? Select all that apply. 1 Oversedation 2 Secretions, coughing, or gagging 3 Loss of airway through total or partial extubation 4 Patient fighting the ventilator 5 Decreased compliance due to pulmonary edema 6 Kinked or compressed tubing

1, 3 Milrinone is a vasodilator. Vasodilation decreases preload and afterload. This medication does not directly affect the heart rate. Vasodilators cause the blood pressure to decrease. Vasodilators will improve cardiac output. Test-Taking Tip: Be alert for details about what you are being asked to do. In this question type, you are asked to select all options that apply to a given situation or patient. All options likely relate to the situation, but only some of the options may relate directly to the situation. Text Reference - p. 1604

A patient is prescribed milrinone. What effects on the patient's hemodynamic parameters should the nurse expect? Select all that apply. 1 Decreased preload 2 Increased heart rate 3 Decreased afterload 4 Increased blood pressure 5 Decreased cardiac output

2 The patient with a ventricular assist device (VAD) may be mobile and require an activity plan such as progressive ambulation. Complete bedrest is not required. The patient will be permitted to do more than move from the bed to a chair twice a day. Activity will be greater than bed rest with bathroom privileges. Text Reference - p. 1613

A patient is recovering from the implantation of a ventricular assist device (VAD). What should the nurse anticipate being prescribed for this patient's activity status? 1 Complete bedrest 2 Progressive ambulation 3 Moving out of bed to a chair twice a day 4 Bedrest with bathroom privileges

6 The patient's airway pressure during expiration is 6 cm H2O. Normally during exhalation, the airway pressure drops to zero and exhalation occurs passively. The pressure in CPAP is delivered continuously during spontaneous breathing to prevent the patient's airway pressure from falling to zero. Therefore, if CPAP is 6 cm H2O, airway pressure during expiration is 6 cm H2O. Text Reference - p. 1621

A patient on mechanical ventilation is receiving a continuous positive airway pressure (CPAP) of 6 cm H2O. What is the patient's airway pressure during expiration? Record your answer using a whole number. ___________________ cm H2O

4 Intrathoracic pressure changes associated with positive pressure ventilation cause a decrease in production of atrial natriuretic peptide. Positive pressure ventilation also decreases cardiac output, which further decreases renal perfusion. A decrease in renal perfusion increases the production of renin, angiotensin and aldosterone, which results in sodium retention. Text Reference - p. 1623

A patient on positive pressure ventilation has increased sodium retention in the body. A decrease in production of which biologic factor may have caused sodium retention in the patient? 1 Renin 2 Angiotensin 3 Aldosterone 4 Atrial natriuretic peptide

1 The patient has a urinary tract infection, which can lead to sepsis. In sepsis, oxygen is not extracted properly at the tissue level, resulting in increased central venous oxygen saturation (ScvO2) mixed venous oxygen saturation (SvO2) measurements. Central venous oxygen saturation (ScvO2) mixed venous oxygen saturation (SvO2) measurements would be low if the patient was experiencing decreased cardiac output or increased oxygen demand. Central venous oxygen saturation (ScvO2) mixed venous oxygen saturation (SvO2) measurements would be between 60% and 80% if the oxygen supply and demand was balanced. Text Reference - p. 1609

A patient with a pulmonary arterial catheter for systolic heart failure is diagnosed with a urinary tract infection (UTI). The last central venous oxygen saturation (ScvO2) mixed venous oxygen saturation (SvO2) measurement was 89%. What should the nurse suspect is occurring with this patient? 1 Sepsis 2 Decreased cardiac output 3 Increased oxygen demand 4 Balanced oxygen supply and deman

2, 4, 6 Negative pressure ventilation is similar to normal ventilation in that expiration is passive and decreased intrathoracic pressures produce inspiration. Negative pressure ventilation uses intermittent subatmospheric pressure around the chest wall. This pressure reduces the intrathoracic pressure during inspiration. Negative pressure ventilation is noninvasive because it does not require an artificial airway. During inspiration, the chest is pulled outward because of the intermittent negative pressure around the chest wall. Negative pressure ventilation uses chambers that encase the chest wall, but not the upper airway. Text Reference - p. 1618

A patient with a spinal cord injury requires negative pressure ventilation. What statements should the nurse say to the patient's caregiver about negative pressure ventilation? Select all that apply. 1 "It is a type of invasive ventilation." 2 "It is similar to the normal ventilation." 3 "It pulls the chest inward during inspiration." 4 "It uses intermittent subatmospheric pressure." 5 "It uses chambers that encase the upper airway." 6 "It reduces intrathoracic pressure during inspiration."

1 The high- and low-pressure alarms are set based on the patient's current status. Since the patient's lowest auscultated systolic blood pressure was 118 mm Hg, the best setting to use would be systolic 100. Because the patient's lowest diastolic blood pressure was 78, the best setting to use would be diastolic 60. The setting of systolic 120 and diastolic 80 may cause the low pressure alarm to go off frequently. The settings of systolic 140, diastolic 80 and systolic 150, diastolic 90 would not be appropriate for low-pressure settings. Text Reference - p. 1606

A patient with an arterial invasive device has the following auscultated blood pressures. What setting should the nurse use for this patient's low pressure alarms? 1 Systolic 100; Diastolic 60 2 Systolic 120; Diastolic 80 3 Systolic 140; Diastolic 80 4 Systolic 150; Diastolic 90

3 A β-adrenergic blocking medication decreases stroke volume. Contractility is reduced by negative inotropes. An example of a negative inotrope is a β-adrenergic blocker. Since an increase in contractility increases stroke volume (SV) and myocardial oxygen requirements, a negative inotrope such as a β-adrenergic blocker will decrease stroke volume (SV). A β-adrenergic blocking medication will decrease rather than increase oxygen use. This medication does not affect myocardial cellular metabolism. Text Reference - p. 1604

A patient with hypertension is prescribed a β-adrenergic blocking medication. What effect should the nurse expect this medication to have on the patient's heart? 1 Increased oxygen use 2 Increased stroke volume 3 Decreased stroke volume 4 Decreased cellular metabolism

3 A central venous oxygen saturation (ScvO2) mixed venous oxygen saturation (SvO2) measurement of 48% is low. Metabolic demand exceeds oxygen supply in conditions that increase muscle movement and metabolic rate, including physiologic states such as seizures. Central venous oxygen saturation (ScvO2) mixed venous oxygen saturation (SvO2) measurements greater than 80% are caused by increased oxygen supply. Even though a low central venous oxygen saturation (ScvO2) mixed venous oxygen saturation (SvO2) measurement is associated with a decreased cardiac output, the patient is not experiencing a health problem such as cardiogenic shock caused by left ventricular pump failure that supports decreased cardiac output as the reason for the low measurement. Central venous oxygen saturation (ScvO2) mixed venous oxygen saturation (SvO2) measurements greater than 80% are caused by decreased oxygen demand. Text Reference - p. 1609

A patient with meningitis and seizures has a pulmonary arterial catheter inserted. The most recent central venous oxygen saturation (ScvO2) mixed venous oxygen saturation (SvO2) measurement is 48%. What should the nurse realize is the reason for this patient's measurement? 1 Increased oxygen supply 2 Decreased cardiac output 3 Increased oxygen demand 4 Decreased oxygen demand

1, 2, 5 In RSI, a sedative and paralytic are administered to the patient. Usually a sedative-hypnotic-amnesic agent like midazolam is used to make the patient unconscious. A rapid-onset opioid like fentanyl is also given to blunt the pain of the procedure. A paralytic drug like succinylcholine is then given to produce skeletal muscle paralysis. Before this, the patient is 100% oxygenated. After intubation, the patient is again 100% oxygenated, and tube placement is confirmed. Auscultation of the chest bilaterally and x-ray are some of the methods of confirming the placement of the tube. During intubation, the endotracheal (ET) tube is inserted through the nose or mouth. While performing RSI, no opening is made in the throat. However, an opening in the throat is made in a procedure called tracheostomy. Text Reference - p. 1614

A patient with severe respiratory distress is brought to the medical facility. The health care provider prescribes rapid-sequence intubation (RSI) to be done. What information should the nurse include when explaining the procedure to the family members? Select all that apply. 1 A sedative and a paralytic medication is administered so that the patient sleeps and does not feel the pain. 2 After intubation, 100% oxygen is given to the patient, and placement of the tube is confirmed. 3 After giving paralytic, an opening is made in the throat through which a tube is introduced. 4 After establishing the opening in the throat, the tube is placed and dressed properly. 5 A tube will be introduced in the patient's throat through the mouth so that ventilation can be established.

120 MAP is calculated by adding the systolic blood pressure to two times the diastolic blood pressure and dividing by three. For this patient that calculation would be 172 + 2(94)/3 = 120 mm Hg. Normal MAP is between 70 and 105 mm Hg. Text Reference - p. 1603

A patient's blood pressure is 172/94 mm Hg. What would the nurse calculate as being this patient's mean arterial pressure (MAP)? Record your answer using a whole number. ____________ mm Hg

4 SIMV stands for synchronized intermittent mandatory ventilation, a mode of ventilation in which the ventilator delivers a preset tidal volume at a preset frequency in synchrony with the patient's spontaneous breathing. Between ventilator-delivered breaths the patient is able to breathe spontaneously, receiving the preset FIO2, but self-regulates the rate and depth of those breaths. Pressure support ventilation (PSV) applies positive pressure only during inspiration. PSV is not used as a sole ventilator support during acute respiratory failure because of the risk of hypoventilation, but it does decrease the work of breathing. Pressure-control inverse ratio ventilation (PC-IRV) sets the ventilation pressure and the ratio of inspiration to expiration to control the patient's breathing. Assist-control ventilation (ACV) or assisted mandatory ventilation (AMV) delivers a preset rate of breaths, but allows the patient to breathe spontaneously, with a preset tidal volume. Text Reference - p. 1620

A patient's family member asks the nurse what SIMV means on the settings of the mechanical ventilator attached to the patient. Which statement best describes this mode of ventilation? 1 "SIMV provides additional inspiratory pressure so that your father does not have to work as hard to breathe, thus enabling better oxygenation and a quicker recovery with fewer complications." 2 "SIMV is a mode that allows the ventilator to totally control your father's breathing. It will prevent him from hyperventilating or hypoventilating, thus ensuring the best oxygenation." 3 "SIMV is a mode that allows your father to breathe on his own, but the ventilator will control how deep a breath he will receive. The ventilator can sense when he wants a breath and it will deliver it." 4 "SIMV is a mode that allows your father to breathe on his own while receiving a preset number of breaths from the ventilator. He can breathe as much or as little as he wants beyond what the ventilator will breathe for him."

1 After insertion and before using the PA catheter, a chest x-ray must be taken to confirm the catheter's position. A hemoglobin level is not needed before using the catheter for fluid administration. Electrolyte levels do not need to be evaluated before using the catheter for fluid administration. A 12-lead electrocardiogram is not needed before using the catheter for fluid administration. Text Reference - p. 1608

A pulmonary artery catheter has just been inserted through a patient's internal jugular vein. What should be done before the catheter is used for fluid administration? 1 Obtain a chest x-ray 2 Draw a hemoglobin level 3 Evaluate electrolyte levels 4 Obtain a 12-lead electrocardiogram

2. The nurse is caring for a patient who has an intraaortic balloon pump (IABP) after a massive heart attack. When assessing the patient, the nurse notices blood backing up into the IABP catheter. In which order should the nurse take the following actions? (Put a comma and a space between each answer choice [A, B, C, D].) a. Confirm that the IABP console has turned off. b. Assess the patient's vital signs and orientation. c. Obtain supplies for insertion of a new IABP catheter. d. Notify the health care provider of the IABP malfunction.

ANS: A, B, D, C Blood in the IABP catheter indicates a possible tear in the balloon. The console should shut off automatically to prevent complications such as air embolism. Next, the nurse will assess the patient and communicate with the health care provider about the patient's assessment and the IABP problem. Finally, supplies for insertion of a new IABP catheter may be needed based on the patient assessment and the decision of the health care provider.

1. When assisting with oral intubation of a patient who is having respiratory distress, in which order will the nurse take these actions? (Put a comma and a space between each answer choice [A, B, C, D, E].) a. Obtain a portable chest-x-ray. b. Position the patient in the supine position. c. Inflate the cuff of the endotracheal tube after insertion. d. Attach an end-tidal CO2 detector to the endotracheal tube. e. Oxygenate the patient with a bag-valve-mask device for several minutes.

ANS: E, B, C, D, A The patient is pre-oxygenated with a bag-valve-mask system for 3 to 5 minutes before intubation and then placed in a supine position. After the intubation, the cuff on the endotracheal tube is inflated to occlude and protect the airway. Tube placement is assessed first with an end-tidal CO2 sensor and then with chest x-ray examination.

1. A patient's vital signs are pulse 87, respirations 24, and BP of 128/64 mm Hg and cardiac output is 4.7 L/min. The patient's stroke volume is _____ mL. (Round to the nearest whole number.)

ANS: 54 Stroke volume = cardiac output/heart rate DIF: Cognitive Level: Understand (comprehension) REF: 1603 TOP: Nursing Process: Assessment MSC: NCLEX: Physiological Integrity

2. The nurse is caring for a patient who has an intraortic balloon pump (IABP) following a massive heart attack. When assessing the patient, the nurse notices blood backing up into the IABP catheter. In which order should the nurse take the following actions? (Put a comma and a space between each answer choice [A, B, C, D].) a. Ensure that the IABP console has turned off. b. Assess the patient's vital signs and orientation. c. Obtain supplies for insertion of a new IABP catheter. d. Notify the health care provider of the IABP malfunction.

ANS: A, B, D, C Blood in the IABP catheter indicates a possible tear in the balloon. The console will shut off automatically to prevent complications such as air embolism. Next, the nurse will assess the patient and communicate with the health care provider about the patient's assessment and the IABP problem. Finally, supplies for insertion of a new IABP catheter may be needed, based on the patient assessment and the decision of the health care provider. DIF: Cognitive Level: Analyze (analysis) REF: 1612 OBJ: Special Questions: Prioritization TOP: Nursing Process: Planning MSC: NCLEX: Physiological Integrity

When assisting with oral intubation of a patient who is having respiratory distress, in which order will the nurse take these actions? (Put a comma and a space between each answer choice [A, B, C, D, E].) a. Obtain a portable chest-x-ray. b. Position the patient in the supine position. c. Inflate the cuff of the endotracheal tube after insertion. d. Attach an end-tidal CO2 detector to the endotracheal tube. e. Oxygenate the patient with a bag-valve-mask device for several minutes.

ANS: E, B, C, D, A The patient is pre-oxygenated with a bag-valve-mask system for 3 to 5 minutes before intubation and then placed in a supine position. After the intubation, the cuff on the endotracheal tube is inflated to occlude and protect the airway. Tube placement is assessed first with an end-tidal CO2 sensor and then with chest x-ray examination.

1. When assisting with oral intubation of a patient who is having respiratory distress, in which order will the nurse take these actions? (Put a comma and a space between each answer choice [A, B, C, D, E].) a. Obtain a portable chest-x-ray. b. Position the patient in the supine position. c. Inflate the cuff of the endotracheal tube after insertion. d. Attach an end-tidal CO2 detector to the endotracheal tube. e. Oxygenate the patient with a bag-valve-mask device for several minutes.

ANS: E, B, C, D, A The patient is pre-oxygenated with a bag-valve-mask system for 3 to 5 minutes before intubation and then placed in a supine position. Following the intubation, the cuff on the endotracheal tube is inflated to occlude and protect the airway. Tube placement is assessed first with an end-tidal CO2 sensor, then with a chest x-ray. DIF: Cognitive Level: Analyze (analysis) REF: 1614 OBJ: Special Questions: Prioritization TOP: Nursing Process: Implementation MSC: NCLEX: Physiological Integrity

22. A patient with respiratory failure has arterial pressure-based cardiac output (APCO) monitoring and is receiving mechanical ventilation with peak end-expiratory pressure (PEEP) of 12 cm H2O. Which information indicates that a change in the ventilator settings may be required? a. The arterial pressure is 90/46. b. The stroke volume is increased. c. The heart rate is 58 beats/minute. d. The stroke volume variation is 12%.

ANS: A The hypotension suggests that the high intrathoracic pressure caused by the PEEP may be decreasing venous return and (potentially) cardiac output. The other assessment data would not be a direct result of PEEP and mechanical ventilation.

35. A patient who is receiving positive pressure ventilation is scheduled for a spontaneous breathing trial (SBT). Which finding by the nurse is most likely to result in postponing the SBT? a. New ST segment elevation is noted on the cardiac monitor. b. Enteral feedings are being given through an orogastric tube. c. Scattered rhonchi are heard when auscultating breath sounds. d. hydromorphone (Dilaudid) is being used to treat postoperative pain.

ANS: A Myocardial ischemia is a contraindication for ventilator weaning. The ST segment elevation is an indication that weaning should be postponed until further investigation and/or treatment for myocardial ischemia can be done. Ventilator weaning can proceed when opioids are used for pain management, abnormal lung sounds are present, or enteral feedings are being used.

36. A patient who is receiving positive pressure ventilation is scheduled for a spontaneous breathing trial (SBT). Which finding by the nurse is most important to discuss with the health care provider before starting the SBT? a. New ST segment elevation is noted on the cardiac monitor. b. Enteral feedings are being given through an orogastric tube. c. Scattered rhonchi are heard when auscultating breath sounds. d. HYDROmorphone (Dilaudid) is being used to treat postoperative pain.

ANS: A Myocardial ischemia is a contraindication for ventilator weaning. The ST segment elevation is an indication that weaning should be postponed until further investigation and/or treatment for myocardial ischemia can be done. The other information will also be shared with the health care provider, but ventilator weaning can proceed when opioids are used for pain management, abnormal lung sounds are present, or enteral feedings are being used. DIF: Cognitive Level: Apply (application) REF: 1626 OBJ: Special Questions: Prioritization TOP: Nursing Process: Assessment MSC: NCLEX: Physiological Integrity

11. Which assessment finding obtained by the nurse when caring for a patient with a right radial arterial line indicates a need for the nurse to take immediate action? a. The right hand is cooler than the left hand. b. The mean arterial pressure (MAP) is 77 mm Hg. c. The system is delivering 3 mL of flush solution per hour. d. The flush bag and tubing were last changed 3 days previously.

ANS: A The change in temperature of the left hand suggests that blood flow to the left hand is impaired. The flush system needs to be changed every 96 hours. A mean arterial pressure (MAP) of 75 mm Hg is normal. Flush systems for hemodynamic monitoring are set up to deliver 3 to 6 mL/hour of flush solution. DIF: Cognitive Level: Apply (application) REF: 1606 TOP: Nursing Process: Assessment MSC: NCLEX: Physiological Integrity

11. Which assessment finding obtained by the nurse when caring for a patient with a right radial arterial line indicates a need for the nurse to take action? a. The right hand feels cooler than the left hand. b. The mean arterial pressure (MAP) is 77 mm Hg. c. The system is delivering 3 mL of flush solution per hour. d. The flush bag and tubing were last changed 2 days previously.

ANS: A The change in temperature of the right hand suggests that blood flow to the right hand is impaired. The flush system needs to be changed every 96 hours. A mean arterial pressure (MAP) of 75 mm Hg is normal. Flush systems for hemodynamic monitoring are set up to deliver 3 to 6 mL/hr of flush solution.

22. A patient with respiratory failure has arterial pressure-based cardiac output (APCO) monitoring and is receiving mechanical ventilation with peak end-expiratory pressure (PEEP) of 12 cm H2O. Which information indicates that a change in the ventilator settings may be required? a. The arterial pressure is 90/46. b. The heart rate is 58 beats/minute. c. The stroke volume is increased. d. The stroke volume variation is 12%.

ANS: A The hypotension suggests that the high intrathoracic pressure caused by the PEEP may be decreasing venous return and (potentially) cardiac output. The other assessment data would not be a direct result of PEEP and mechanical ventilation. DIF: Cognitive Level: Apply (application) REF: 1622-1624 TOP: Nursing Process: Evaluation MSC: NCLEX: Physiological Integrity

33. A patient who is orally intubated and receiving mechanical ventilation is anxious and is "fighting" the ventilator. Which action should the nurse take next? a. Verbally coach the patient to breathe with the ventilator. b. Sedate the patient with the ordered PRN lorazepam (Ativan). c. Manually ventilate the patient with a bag-valve-mask device. d. Increase the rate for the ordered propofol (Diprivan) infusion.

ANS: A The initial response by the nurse should be to try to decrease the patient's anxiety by coaching the patient about how to coordinate respirations with the ventilator. The other actions may also be helpful if the verbal coaching is ineffective in reducing the patient's anxiety. DIF: Cognitive Level: Apply (application) REF: 1623 OBJ: Special Questions: Prioritization TOP: Nursing Process: Implementation MSC: NCLEX: Physiological Integrity

4. After surgery for an abdominal aortic aneurysm, a patient's central venous pressure (CVP) monitor indicates low pressures. Which action should the nurse take? a. Administer IV diuretic medications. b. Increase the IV fluid infusion per protocol. c. Increase the infusion rate of IV vasodilators. d. Elevate the head of the patient's bed to 45 degrees.

ANS: B A low CVP indicates hypovolemia and a need for an increase in the infusion rate. Diuretic administration will contribute to hypovolemia and elevation of the head or increasing vasodilators may decrease cerebral perfusion.

4. Following surgery for an abdominal aortic aneurysm, a patient's central venous pressure (CVP) monitor indicates low pressures. Which action is a priority for the nurse to take? a. Administer IV diuretic medications. b. Increase the IV fluid infusion per protocol. c. Document the CVP and continue to monitor. d. Elevate the head of the patient's bed to 45 degrees.

ANS: B A low CVP indicates hypovolemia and a need for an increase in the infusion rate. Diuretic administration will contribute to hypovolemia and elevation of the head may decrease cerebral perfusion. Documentation and continued monitoring is an inadequate response to the low CVP. DIF: Cognitive Level: Apply (application) REF: 1609 TOP: Nursing Process: Planning MSC: NCLEX: Physiological Integrity

1. A patient who has been in the intensive care unit for 4 days has disturbed sensory perception from sleep deprivation. Which action should the nurse include in the plan of care? a. Administer prescribed sedatives or opioids at bedtime to promote sleep. b. Cluster nursing activities so that the patient has uninterrupted rest periods. c. Silence the alarms on the cardiac monitors to allow 30- to 40-minute naps. d. Eliminate assessments between 2200 and 0600 to allow uninterrupted sleep.

ANS: B Clustering nursing activities and providing uninterrupted rest periods will minimize sleep-cycle disruption. Sedative and opioid medications tend to decrease the amount of rapid eye movement (REM) sleep and can contribute to sleep disturbance and disturbed sensory perception. Silencing the alarms on the cardiac monitors would be unsafe in a critically ill patient, as would discontinuing all assessments during the night.

1. A 68-year-old patient has been in the intensive care unit for 4 days and has a nursing diagnosis of disturbed sensory perception related to sleep deprivation. Which action should the nurse include in the plan of care? a. Administer prescribed sedatives or opioids at bedtime to promote sleep. b. Cluster nursing activities so that the patient has uninterrupted rest periods. c. Silence the alarms on the cardiac monitors to allow 30- to 40-minute naps. d. Eliminate assessments between 0100 and 0600 to allow uninterrupted sleep.

ANS: B Clustering nursing activities and providing uninterrupted rest periods will minimize sleep-cycle disruption. Sedative and opioid medications tend to decrease the amount of rapid eye movement (REM) sleep and can contribute to sleep disturbance and disturbed sensory perception. Silencing the alarms on the cardiac monitors would be unsafe in a critically ill patient, as would discontinuing assessments during the night. DIF: Cognitive Level: Apply (application) REF: 1601 TOP: Nursing Process: Planning MSC: NCLEX: Psychosocial Integrity

12. The central venous oxygen saturation (ScvO2) is decreasing in a patient who has severe pancreatitis. To determine the possible cause of the decreased ScvO2 , the nurse assesses the patient's a. lipase level. c. urinary output. b. temperature. d. body mass index.

ANS: B Elevated temperature increases metabolic demands and O2 use by tissues, resulting in a drop in O2 saturation of central venous blood. Information about the patient's body mass index, urinary output, and lipase will not help in determining the cause of the patient's drop in ScvO2

12. The central venous oxygen saturation (ScvO2) is decreasing in a patient who has severe pancreatitis. To determine the possible cause of the decreased ScvO2, the nurse assesses the patient's a. lipase. b. temperature. c. urinary output. d. body mass index.

ANS: B Elevated temperature increases metabolic demands and oxygen use by tissues, resulting in a drop in oxygen saturation of central venous blood. Information about the patient's body mass index, urinary output, and lipase will not help in determining the cause of the patient's drop in ScvO2. DIF: Cognitive Level: Apply (application) REF: 1609 TOP: Nursing Process: Assessment MSC: NCLEX: Physiological Integrity

16. To verify the correct placement of an oral endotracheal tube (ET) after insertion, the best initial action by the nurse is to a. obtain a portable chest x-ray. b. use an end-tidal CO2 monitor. c. auscultate for bilateral breath sounds. d. observe for symmetrical chest movement.

ANS: B End-tidal CO2 monitors are currently recommended for rapid verification of ET placement. Auscultation for bilateral breath sounds and checking chest expansion are also used, but they are not as accurate as end-tidal CO2 monitoring. A chest x-ray confirms the placement but is done after the tube is secured.

3. While close family members are visiting, a patient has a respiratory arrest, and resuscitation is started. Which action by the nurse is best? a. Tell the family members that watching the resuscitation will be very stressful. b. Ask family members if they wish to remain in the room during the resuscitation. c. Take the family members quickly out of the patient room and remain with them. d. Assign a staff member to wait with family members just outside the patient room.

ANS: B Evidence indicates that many family members want the option of remaining in the room during procedures such as cardiopulmonary resuscitation (CPR) and that this decreases anxiety and facilitates grieving. The other options may be appropriate if the family decides not to remain with the patient.

6. The intensive care unit (ICU) nurse educator determines that teaching a new staff nurse about arterial pressure monitoring has been effective when the nurse a. balances and calibrates the monitoring equipment every 2 hours. b. positions the zero-reference stopcock line level with the phlebostatic axis. c. ensures that the patient is supine with the head of the bed flat for all readings. d. rechecks the location of the phlebostatic axis with changes in the patient's position.

ANS: B For accurate measurement of pressures, the zero-reference level should be at the phlebostatic axis. There is no need to rebalance and recalibrate monitoring equipment every 2 hours. Accurate hemodynamic readings are possible with the patient's head raised to 45 degrees or in the prone position. The anatomic position of the phlebostatic axis does not change when patients are repositioned.

6. The intensive care unit (ICU) nurse educator will determine that teaching about arterial pressure monitoring for a new staff nurse has been effective when the nurse a. balances and calibrates the monitoring equipment every 2 hours. b. positions the zero-reference stopcock line level with the phlebostatic axis. c. ensures that the patient is supine with the head of the bed flat for all readings. d. rechecks the location of the phlebostatic axis when changing the patient's position.

ANS: B For accurate measurement of pressures, the zero-reference level should be at the phlebostatic axis. There is no need to rebalance and recalibrate monitoring equipment hourly. Accurate hemodynamic readings are possible with the patient's head raised to 45 degrees or in the prone position. The anatomic position of the phlebostatic axis does not change when patients are repositioned. DIF: Cognitive Level: Apply (application) REF: 1605 TOP: Nursing Process: Evaluation MSC: NCLEX: Safe and Effective Care Environment

14. The nurse is caring for a patient who has an intraaortic balloon pump in place. Which action should be included in the plan of care? a. Avoid the use of anticoagulant medications. b. Measure the patient's urinary output every hour. c. Provide passive range of motion for all extremities. d. Position the patient supine with head flat at all times.

ANS: B Monitoring urine output will help determine whether the patient's cardiac output has improved and also help monitor for balloon displacement blocking the renal arteries. The head of the bed can be elevated up to 30 degrees. Heparin is used to prevent thrombus formation. Limited movement is allowed for the extremity with the balloon insertion site to prevent displacement of the balloon.

28. The nurse is caring for a patient who has an arterial catheter in the left radial artery for arterial pressure-based cardiac output (APCO) monitoring. Which information obtained by the nurse requires a report to the health care provider? a. The patient has a positive Allen test result. b. There is redness at the catheter insertion site. c. The mean arterial pressure (MAP) is 86 mm Hg. d. The dicrotic notch is visible in the arterial waveform.

ANS: B Redness at the catheter insertion site indicates possible infection. The Allen test is performed before arterial line insertion, and a positive test result indicates normal ulnar artery perfusion. A MAP of 86 mm Hg is normal, and the dicrotic notch is normally present on the arterial waveform.

29. When caring for a patient who has an arterial catheter in the left radial artery for arterial pressure-based cardiac output (APCO) monitoring, which information obtained by the nurse is most important to report to the health care provider? a. The patient has a positive Allen test. b. There is redness at the catheter insertion site. c. The mean arterial pressure (MAP) is 86 mm Hg. d. The dicrotic notch is visible in the arterial waveform.

ANS: B Redness at the catheter insertion site indicates possible infection. The Allen test is performed before arterial line insertion, and a positive test indicates normal ulnar artery perfusion. A MAP of 86 is normal and the dicrotic notch is normally present on the arterial waveform. DIF: Cognitive Level: Apply (application) REF: 1606 OBJ: Special Questions: Prioritization TOP: Nursing Process: Assessment MSC: NCLEX: Physiological Integrity

3. While family members are visiting, a patient has a respiratory arrest and is being resuscitated. Which action by the nurse is best? a. Tell the family members that watching the resuscitation will be very stressful. b. Ask family members if they wish to remain in the room during the resuscitation. c. Take the family members quickly out of the patient room and remain with them. d. Assign a staff member to wait with family members just outside the patient room.

ANS: B Research indicates that family members want the option of remaining in the room during procedures such as cardiopulmonary resuscitation (CPR) and that this decreases anxiety and facilitates grieving. The other options may be appropriate if the family decides not to remain with the patient. DIF: Cognitive Level: Apply (application) REF: 1602 TOP: Nursing Process: Implementation MSC: NCLEX: Psychosocial Integrity

2. Which hemodynamic parameter best reflects the effectiveness of drugs that the nurse gives to reduce a patient's left ventricular afterload? a. Mean arterial pressure (MAP) b. Systemic vascular resistance (SVR) c. Pulmonary vascular resistance (PVR) d. Pulmonary artery wedge pressure (PAWP)

ANS: B SVR reflects the resistance to ventricular ejection, or afterload. The other parameters may be monitored but do not reflect afterload as directly.

2. Which hemodynamic parameter is most appropriate for the nurse to monitor to determine the effectiveness of medications given to a patient to reduce left ventricular afterload? a. Mean arterial pressure (MAP) b. Systemic vascular resistance (SVR) c. Pulmonary vascular resistance (PVR) d. Pulmonary artery wedge pressure (PAWP)

ANS: B Systemic vascular resistance reflects the resistance to ventricular ejection, or afterload. The other parameters will be monitored, but do not reflect afterload as directly.

33. The nurse educator is evaluating the performance of a new registered nurse (RN) who is providing care to a patient who is receiving mechanical ventilation with 15 cm H2O of peak end- expiratory pressure (PEEP). Which action indicates that the new RN is safe? a. The RN plans to suction the patient every 1 to 2 hours. b. The RN uses a closed-suction technique to suction the patient. c. The RN tapes the connection between the ventilator tubing and the ET. d. The RN changes the ventilator circuit tubing routinely every 48 hours.

ANS: B The closed-suction technique is used when patients require high levels of PEEP (>10 cm H2O) to prevent the loss of PEEP that occurs when disconnecting the patient from the ventilator. Suctioning should not be scheduled routinely, but it should be done only when patient assessment data indicate the need for suctioning. Taping connections between the ET and ventilator tubing would restrict the ability of the tubing to swivel in response to patient repositioning. Ventilator tubing changes increase the risk for ventilator-associated pneumonia and are not indicated routinely.

34. The nurse educator is evaluating the performance of a new registered nurse (RN) who is providing care to a patient who is receiving mechanical ventilation with 15 cm H2O of peak end-expiratory pressure (PEEP). Which action indicates that the new RN is safe? a. The RN plans to suction the patient every 1 to 2 hours. b. The RN uses a closed-suction technique to suction the patient. c. The RN tapes connection between the ventilator tubing and the ET. d. The RN changes the ventilator circuit tubing routinely every 48 hours.

ANS: B The closed-suction technique is used when patients require high levels of PEEP (>10 cm H2O) to prevent the loss of PEEP that occurs when disconnecting the patient from the ventilator. Suctioning should not be scheduled routinely, but it should be done only when patient assessment data indicate the need for suctioning. Taping connections between the ET and the ventilator tubing would restrict the ability of the tubing to swivel in response to patient repositioning. Ventilator tubing changes increase the risk for ventilator-associated pneumonia (VAP) and are not indicated routinely. DIF: Cognitive Level: Apply (application) REF: 1616 OBJ: Special Questions: Delegation TOP: Nursing Process: Implementation MSC: NCLEX: Safe and Effective Care Environment

26. When evaluating a patient with a central venous catheter, the nurse observes that the insertion site is red and tender to touch and the patient's temperature is 101.8° F. What should the nurse plan to do next? a. Give analgesics and antibiotics as ordered. b. Discontinue the catheter and culture the tip. c. Change the flush system and monitor the site. d. Check the site more frequently for any swelling.

ANS: B The information indicates that the patient has a local and systemic infection caused by the catheter, and the catheter should be discontinued. Changing the flush system, giving analgesics, and continued monitoring will not help prevent or treat the infection. Administration of antibiotics is appropriate, but the line should still be discontinued to avoid further complications such as endocarditis. DIF: Cognitive Level: Apply (application) REF: 1611 TOP: Nursing Process: Planning MSC: NCLEX: Physiological Integrity

26. An 81-yr-old patient who has been in the intensive care unit (ICU) for a week is now stable and transfer to the progressive care unit is planned. On rounds, the nurse notices that the patient has new onset confusion. The nurse will plan to a. give PRN lorazepam (Ativan) and cancel the transfer. b. inform the receiving nurse and then transfer the patient. c. notify the health care provider and postpone the transfer. d. obtain an order for restraints as needed and transfer the patient.

ANS: B The patient's history and symptoms most likely indicate delirium associated with the sleep deprivation and sensory overload in the ICU environment. Informing the receiving nurse and transferring the patient is appropriate. Postponing the transfer is likely to prolong the delirium. Benzodiazepines and restraints contribute to delirium and agitation.

27. An 81-year-old patient who has been in the intensive care unit (ICU) for a week is now stable and transfer to the progressive care unit is planned. On rounds, the nurse notices that the patient has new onset confusion. The nurse will plan to a. give PRN lorazepam (Ativan) and cancel the transfer. b. inform the receiving nurse and then transfer the patient. c. notify the health care provider and postpone the transfer. d. obtain an order for restraints as needed and transfer the patient.

ANS: B The patient's history and symptoms most likely indicate delirium associated with the sleep deprivation and sensory overload in the ICU environment. Informing the receiving nurse and transferring the patient is appropriate. Postponing the transfer is likely to prolong the delirium. Benzodiazepines and restraints contribute to delirium and agitation. DIF: Cognitive Level: Apply (application) REF: 1601 TOP: Nursing Process: Planning MSC: NCLEX: Psychosocial Integrity

13. An intraaortic balloon pump (IABP) is being used for a patient who is in cardiogenic shock. Which assessment data indicate to the nurse that the goals of treatment with the IABP are being met? a. Urine output of 25 mL/hr b. Heart rate of 110 beats/minute c. Cardiac output (CO) of 5 L/min d. Stroke volume (SV) of 40 mL/beat

ANS: C A CO of 5 L/min is normal and indicates that the IABP has been successful in treating the shock. The low SV signifies continued cardiogenic shock. The tachycardia and low urine output also suggest continued cardiogenic shock.

13. An intraaortic balloon pump (IABP) is being used for a patient who is in cardiogenic shock. Which assessment data indicate to the nurse that the goals of treatment with the IABP are being met? a. Urine output of 25 mL/hr b. Heart rate of 110 beats/minute c. Cardiac output (CO) of 5 L/min d. Stroke volume (SV) of 40 mL/beat

ANS: C A CO of 5 L/min is normal and indicates that the IABP has been successful in treating the shock. The low SV signifies continued cardiogenic shock. The tachycardia and low urine output also suggest continued cardiogenic shock. DIF: Cognitive Level: Apply (application) REF: 1603 TOP: Nursing Process: Evaluation MSC: NCLEX: Physiological Integrity

20. The nurse notes thick, white secretions in the endotracheal tube (ET) of a patient who is receiving mechanical ventilation. Which intervention will most directly treat this finding? a. Reposition the patient every 1 to 2 hours. b. Increase suctioning frequency to every hour. c. Add additional water to the patient's enteral feedings. d. Instill 5 mL of sterile saline into the ET before suctioning.

ANS: C Because the patient's secretions are thick, better hydration is indicated. Suctioning every hour without any specific evidence for the need will increase the incidence of mucosal trauma and would not address the etiology of the ineffective airway clearance. Instillation of saline does not liquefy secretions and may decrease the SpO2 . Repositioning the patient is appropriate but will not decrease the thickness of secretions.

20. The nurse notes thick, white secretions in the endotracheal tube (ET) of a patient who is receiving mechanical ventilation. Which intervention will be most effective in addressing this problem? a. Increase suctioning to every hour. b. Reposition the patient every 1 to 2 hours. c. Add additional water to the patient's enteral feedings. d. Instill 5 mL of sterile saline into the ET before suctioning.

ANS: C Because the patient's secretions are thick, better hydration is indicated. Suctioning every hour without any specific evidence for the need will increase the incidence of mucosal trauma and would not address the etiology of the ineffective airway clearance. Instillation of saline does not liquefy secretions and may decrease the SpO2. Repositioning the patient is appropriate but will not decrease the thickness of secretions. DIF: Cognitive Level: Apply (application) REF: 1617 TOP: Nursing Process: Implementation MSC: NCLEX: Physiological Integrity

18. The nurse notes premature ventricular contractions (PVCs) while suctioning a patient's endotracheal tube. Which action by the nurse is a priority? a. Decrease the suction pressure to 80 mm Hg. b. Document the dysrhythmia in the patient's chart. c. Stop and ventilate the patient with 100% oxygen. d. Give antidysrhythmic medications per protocol.

ANS: C Dysrhythmias during suctioning may indicate hypoxemia or sympathetic nervous system stimulation. The nurse should stop suctioning and ventilate the patient with 100% oxygen. Lowering the suction pressure will decrease the effectiveness of suctioning without improving the hypoxemia. Because the PVCs occurred during suctioning, there is no need for antidysrhythmic medications (which may have adverse effects) unless they recur when the suctioning is stopped and patient is well oxygenated. DIF: Cognitive Level: Apply (application) REF: 1616 OBJ: Special Questions: Prioritization TOP: Nursing Process: Implementation MSC: NCLEX: Physiological Integrity

27. The family members of a patient who has been admitted to the intensive care unit (ICU) with multiple traumatic injuries have just arrived in the ICU waiting room. Which action should the nurse take first? a. Explain ICU visitation policies and encourage family visits. b. Escort the family from the waiting room to the patient's bedside. c. Describe the patient's injuries and the care that is being provided. d. Invite the family to participate in an interprofessional care conference.

ANS: C Lack of information is a major source of anxiety for family members and should be addressed first. Family members should be prepared for the patient's appearance and the ICU environment before visiting the patient for the first time. ICU visiting should be individualized to each patient and family rather than being dictated by rigid visitation policies. Inviting the family to participate in a multidisciplinary conference is appropriate but should not be the initial action by the nurse.

28. The family members of a patient who has just been admitted to the intensive care unit (ICU) with multiple traumatic injuries have just arrived in the ICU waiting room. Which action should the nurse take next? a. Explain ICU visitation policies and encourage family visits. b. Immediately take the family members to the patient's bedside. c. Describe the patient's injuries and the care that is being provided. d. Invite the family to participate in a multidisciplinary care conference.

ANS: C Lack of information is a major source of anxiety for family members and should be addressed first. Family members should be prepared for the patient's appearance and the ICU environment before visiting the patient for the first time. ICU visiting should be individualized to each patient and family rather than being dictated by rigid visitation policies. Inviting the family to participate in a multidisciplinary conference is appropriate but should not be the initial action by the nurse. DIF: Cognitive Level: Apply (application) REF: 1602 OBJ: Special Questions: Prioritization TOP: Nursing Process: Implementation MSC: NCLEX: Psychosocial Integrity

14. The nurse is caring for a patient who has an intraaortic balloon pump in place. Which action should be included in the plan of care? a. Position the patient supine at all times. b. Avoid the use of anticoagulant medications. c. Measure the patient's urinary output every hour. d. Provide passive range of motion for all extremities.

ANS: C Monitoring urine output will help determine whether the patient's cardiac output has improved and also help monitor for balloon displacement. The head of the bed can be elevated up to 30 degrees. Heparin is used to prevent thrombus formation. Limited movement is allowed for the extremity with the balloon insertion site to prevent displacement of the balloon. DIF: Cognitive Level: Apply (application) REF: 1613 TOP: Nursing Process: Planning MSC: NCLEX: Physiological Integrity

5. When caring for a patient with pulmonary hypertension, which parameter will the nurse use to directly evaluate the effectiveness of the treatment? a. Central venous pressure (CVP) b. Systemic vascular resistance (SVR) c. Pulmonary vascular resistance (PVR) d. Pulmonary artery wedge pressure (PAWP)

ANS: C PVR is a major contributor to pulmonary hypertension, and a decrease would indicate that pulmonary hypertension was improving. The other parameters may also be monitored but do not directly assess for pulmonary hypertension.

5. When caring for a patient with pulmonary hypertension, which parameter is most appropriate for the nurse to monitor to evaluate the effectiveness of the treatment? a. Central venous pressure (CVP) b. Systemic vascular resistance (SVR) c. Pulmonary vascular resistance (PVR) d. Pulmonary artery wedge pressure (PAWP)

ANS: C PVR is a major contributor to pulmonary hypertension, and a decrease would indicate that pulmonary hypertension was improving. The other parameters also may be monitored but do not directly assess for pulmonary hypertension. DIF: Cognitive Level: Apply (application) REF: 1603-1604 TOP: Nursing Process: Evaluation MSC: NCLEX: Physiological Integrity

23. A nurse is weaning a 68-kg patient who has chronic obstructive pulmonary disease (COPD) from mechanical ventilation. Which patient assessment finding indicates that the weaning protocol should be stopped? a. The patient's heart rate is 97 beats/min. b. The patient's oxygen saturation is 93%. c. The patient respiratory rate is 32 breaths/min. d. The patient's spontaneous tidal volume is 450 mL.

ANS: C Tachypnea is a sign that the patient's work of breathing is too high to allow weaning to proceed. The patient's heart rate is within normal limits, but the nurse should continue to monitor it. An O2 saturation of 93% is acceptable for a patient with COPD. A spontaneous tidal volume of 450 mL is within the acceptable range.

23. A nurse is weaning a 68-kg male patient who has chronic obstructive pulmonary disease (COPD) from mechanical ventilation. Which patient assessment finding indicates that the weaning protocol should be stopped? a. The patient's heart rate is 97 beats/min. b. The patient's oxygen saturation is 93%. c. The patient respiratory rate is 32 breaths/min. d. The patient's spontaneous tidal volume is 450 mL.

ANS: C Tachypnea is a sign that the patient's work of breathing is too high to allow weaning to proceed. The patient's heart rate is within normal limits, although the nurse should continue to monitor it. An oxygen saturation of 93% is acceptable for a patient with COPD. A spontaneous tidal volume of 450 mL is within the acceptable range. DIF: Cognitive Level: Apply (application) REF: 1627 TOP: Nursing Process: Evaluation MSC: NCLEX: Physiological Integrity

36. After change-of-shift report on a ventilator weaning unit, which patient should the nurse assess first? a. Patient who failed a spontaneous breathing trial and has been placed in a rest mode on the ventilator b. Patient who is intubated and has continuous partial pressure end-tidal CO2 (PETCO2) monitoring c. Patient who was successfully weaned and extubated 4 hours ago and has no urine output for the last 6 hours d. Patient with a central venous O2 saturation (ScvO2) of 69% while on bilevel positive airway pressure (BiPAP)

ANS: C The decreased urine output may indicate acute kidney injury or that the patient's cardiac output and perfusion of vital organs have decreased. Any of these causes would require rapid action. The data about the other patients indicate that their conditions are stable and do not require immediate assessment or changes in their care. Continuous PETCO2 monitoring is frequently used when patients are intubated. The rest mode should be used to allow patient recovery after a failed SBT, and an ScvO2 of 69% is within normal limits.

19. Which assessment finding obtained by the nurse when caring for a patient receiving mechanical ventilation indicates the need for suctioning? a. The patient was last suctioned 6 hours ago. b. The patient's oxygen saturation drops to 93%. c. The patient's respiratory rate is 32 breaths/min. d. The patient has occasional audible expiratory wheezes.

ANS: C The increase in respiratory rate indicates that the patient may have decreased airway clearance and requires suctioning. Suctioning is done when patient assessment data indicate that it is needed and not on a scheduled basis. Occasional expiratory wheezes do not indicate poor airway clearance, and suctioning the patient may induce bronchospasm and increase wheezing. An O2 saturation of 93% is acceptable and does not suggest that immediate suctioning is needed.

19. Which assessment finding obtained by the nurse when caring for a patient receiving mechanical ventilation indicates the need for suctioning? a. The patient's oxygen saturation is 93%. b. The patient was last suctioned 6 hours ago. c. The patient's respiratory rate is 32 breaths/minute. d. The patient has occasional audible expiratory wheezes.

ANS: C The increase in respiratory rate indicates that the patient may have decreased airway clearance and requires suctioning. Suctioning is done when patient assessment data indicate that it is needed, not on a scheduled basis. Occasional expiratory wheezes do not indicate poor airway clearance, and suctioning the patient may induce bronchospasm and increase wheezing. An oxygen saturation of 93% is acceptable and does not suggest that immediate suctioning is needed. DIF: Cognitive Level: Apply (application) REF: 1616 TOP: Nursing Process: Assessment MSC: NCLEX: Physiological Integrity

8. Which action should the nurse take when the low pressure alarm sounds for a patient who has an arterial line in the left radial artery? a. Fast flush the arterial line. b. Check the left hand for pallor. c. Assess for cardiac dysrhythmias. d. Re-zero the monitoring equipment.

ANS: C The low pressure alarm indicates a drop in the patient's blood pressure, which may be caused by cardiac dysrhythmias. There is no indication to re-zero the equipment. Pallor of the left hand would be caused by occlusion of the radial artery by the arterial catheter, not by low pressure. There is no indication of a need for flushing the line.

8. Which action is a priority for the nurse to take when the low pressure alarm sounds for a patient who has an arterial line in the left radial artery? a. Fast flush the arterial line. b. Check the left hand for pallor. c. Assess for cardiac dysrhythmias. d. Rezero the monitoring equipment.

ANS: C The low pressure alarm indicates a drop in the patient's blood pressure, which may be caused by cardiac dysrhythmias. There is no indication to rezero the equipment. Pallor of the left hand would be caused by occlusion of the radial artery by the arterial catheter, not by low pressure. There is no indication of a need for flushing the line. DIF: Cognitive Level: Apply (application) REF: 1606 OBJ: Special Questions: Prioritization TOP: Nursing Process: Implementation MSC: NCLEX: Physiological Integrity

30. The nurse notes that a patient's endotracheal tube (ET), which was at the 22-cm mark, is now at the 25-cm mark, and the patient is anxious and restless. Which action should the nurse take next? a. Check the O2 saturation. b. Offer reassurance to the patient. c. Listen to the patient's breath sounds. d. Notify the patient's health care provider.

ANS: C The nurse should first determine whether the ET tube has been displaced into the right mainstem bronchus by listening for unilateral breath sounds. If so, assistance will be needed to reposition the tube immediately. The other actions are also appropriate, but detection and correction of tube malposition are the most critical actions.

31. The nurse notes that a patient's endotracheal tube (ET), which was at the 22-cm mark, is now at the 25-cm mark and the patient is anxious and restless. Which action should the nurse take next? a. Offer reassurance to the patient. b. Bag the patient at an FIO2 of 100%. c. Listen to the patient's breath sounds. d. Notify the patient's health care provider.

ANS: C The nurse should first determine whether the ET tube has been displaced into the right mainstem bronchus by listening for unilateral breath sounds. If so, assistance will be needed to reposition the tube immediately. The other actions are also appropriate, but detection and correction of tube malposition are the most critical actions. DIF: Cognitive Level: Apply (application) REF: 1614 OBJ: Special Questions: Prioritization TOP: Nursing Process: Implementation MSC: NCLEX: Physiological Integrity

24. The nurse is caring for a patient receiving a continuous norepinephrine (Levophed) IV infusion. Which patient assessment finding indicates that the infusion rate may need to be adjusted? a. Heart rate is 58 beats/minute. b. Mean arterial pressure (MAP) is 56 mm Hg. c. Systemic vascular resistance (SVR) is elevated. d. Pulmonary artery wedge pressure (PAWP) is low.

ANS: C Vasoconstrictors such as norepinephrine (Levophed) will increase SVR, and this will increase the work of the heart and decrease peripheral perfusion. The infusion rate may need to be decreased. Bradycardia, hypotension (MAP of 56 mm Hg), and low PAWP are not associated with norepinephrine infusion. DIF: Cognitive Level: Apply (application) REF: 1604 TOP: Nursing Process: Evaluation MSC: NCLEX: Physiological Integrity

24. The nurse is caring for a patient receiving a continuous norepinephrine IV infusion. Which patient assessment finding indicates that the infusion rate may need to be adjusted? a. Heart rate is slow at 58 beats/min. b. Mean arterial pressure (MAP) is 56 mm Hg. c. Systemic vascular resistance (SVR) is elevated. d. Pulmonary artery wedge pressure (PAWP) is low.

ANS: C Vasoconstrictors such as norepinephrine will increase SVR, and this will increase the work of the heart and decrease peripheral perfusion. The infusion rate may need to be decreased. Bradycardia, hypotension (MAP of 56 mm Hg), and low PAWP are not associated with norepinephrine infusion.

15. While waiting for heart transplantation, a patient with severe cardiomyopathy has a ventricular assist device (VAD) implanted. When planning care for this patient, the nurse should anticipate a. preparing the patient for a permanent VAD. b. administering immunosuppressive medications. c. teaching the patient the reason for complete bed rest. d. monitoring the surgical incision for signs of infection.

ANS: D The insertion site for the VAD provides a source for transmission of infection to the circulatory system and requires frequent monitoring. Patients with VADs are able to have some mobility and may not be on bed rest. The VAD is a bridge to transplantation, not a permanent device. Immunosuppression is not necessary for nonbiologic devices such as the VAD.

29. The nurse responds to a ventilator alarm and finds the patient lying in bed gasping and holding the endotracheal tube (ET) in her hand. Which action should the nurse take next? a. Activate the rapid response team. b. Provide reassurance to the patient. c. Call the health care provider to reinsert the tube. d. Manually ventilate the patient with 100% oxygen.

ANS: D The nurse should ensure maximal patient oxygenation by manually ventilating with a bag-valve-mask system. Offering reassurance to the patient, notifying the health care provider about the need to reinsert the tube, and activating the rapid response team are also appropriate after the nurse has stabilized the patient's oxygenation.

9. Which nursing action is needed when preparing to assist with the insertion of a pulmonary artery catheter? a. Determine if the cardiac troponin level is elevated. b. Auscultate heart sounds before and during insertion. c. Place the patient on NPO status before the procedure. d. Attach cardiac monitoring leads before the procedure.

ANS: D Dysrhythmias can occur as the catheter is floated through the right atrium and ventricle, and it is important for the nurse to monitor for these during insertion. Pulmonary artery catheter insertion does not require anesthesia, and the patient will not need to be NPO. Changes in cardiac troponin or heart and breath sounds are not expected during pulmonary artery catheter insertion.

9. Which action will the nurse need to do when preparing to assist with the insertion of a pulmonary artery catheter? a. Determine if the cardiac troponin level is elevated. b. Auscultate heart and breath sounds during insertion. c. Place the patient on NPO status before the procedure. d. Attach cardiac monitoring leads before the procedure.

ANS: D Dysrhythmias can occur as the catheter is floated through the right atrium and ventricle, and it is important for the nurse to monitor for these during insertion. Pulmonary artery catheter insertion does not require anesthesia, and the patient will not need to be NPO. Changes in cardiac troponin or heart and breath sounds are not expected during pulmonary artery catheter insertion. DIF: Cognitive Level: Apply (application) REF: 1608 TOP: Nursing Process: Planning MSC: NCLEX: Physiological Integrity

18. The nurse notes premature ventricular contractions (PVCs) while suctioning a patient's endotracheal tube. Which next action by the nurse is indicated? a. Plan to suction the patient more frequently. b. Decrease the suction pressure to 80 mm Hg. c. Give antidysrhythmic medications per protocol. d. Stop and ventilate the patient with 100% oxygen.

ANS: D Dysrhythmias during suctioning may indicate hypoxemia or sympathetic nervous system stimulation. The nurse should stop suctioning and ventilate the patient with 100% O2 . There is no indication that more frequent suctioning is needed. Lowering the suction pressure will decrease the effectiveness of suctioning without improving the hypoxemia. Because the PVCs occurred during suctioning, there is no need for antidysrhythmic medications (which may have adverse effects) unless they recur when the suctioning is stopped and patient is well oxygenated.

16. To verify the correct placement of an oral endotracheal tube (ET) after insertion, the best initial action by the nurse is to a. auscultate for the presence of bilateral breath sounds. b. obtain a portable chest x-ray to check tube placement. c. observe the chest for symmetric chest movement with ventilation. d. use an end-tidal CO2 monitor to check for placement in the trachea.

ANS: D End-tidal CO2 monitors are currently recommended for rapid verification of ET placement. Auscultation for bilateral breath sounds and checking chest expansion are also used, but they are not as accurate as end-tidal CO2 monitoring. A chest x-ray confirms the placement but is done after the tube is secured. DIF: Cognitive Level: Apply (application) REF: 1614-1615 TOP: Nursing Process: Evaluation MSC: NCLEX: Physiological Integrity

34. The nurse is caring for a patient with a subarachnoid hemorrhage who is intubated and placed on a mechanical ventilator with 10 cm H2O of peak end-expiratory pressure (PEEP). When monitoring the patient, the nurse will need to notify the health care provider immediately if the patient develops a. O2 saturation of 93%. b. green nasogastric tube drainage. c. respirations of 20 breaths/minute. d. increased jugular venous distention.

ANS: D Increases in jugular venous distention in a patient with a subarachnoid hemorrhage may indicate an increase in intracranial pressure (ICP) and that the PEEP setting is too high for this patient. A respiratory rate of 20, O2 saturation of 93%, and green nasogastric tube drainage are within normal limits.

32. The nurse educator is evaluating the care that a new registered nurse (RN) provides to a patient receiving mechanical ventilation. Which action by the new RN indicates the need for more education? a. The RN increases the FIO2 to 100% before suctioning. b. The RN secures a bite block in place using adhesive tape. c. The RN asks for assistance to reposition the endotracheal tube. d. The RN positions the patient with the head of bed at 10 degrees.

ANS: D The head of the patient's bed should be positioned at 30 to 45 degrees to prevent ventilator-associated pneumonia. The other actions by the new RN are appropriate. DIF: Cognitive Level: Apply (application) REF: 1623 OBJ: Special Questions: Delegation TOP: Nursing Process: Evaluation MSC: NCLEX: Safe and Effective Care Environment

35. The nurse is caring for a patient with a subarachnoid hemorrhage who is intubated and placed on a mechanical ventilator with 10 cm H2O of peak end-expiratory pressure (PEEP). When monitoring the patient, the nurse will need to notify the health care provider immediately if the patient develops a. oxygen saturation of 93%. b. respirations of 20 breaths/minute. c. green nasogastric tube drainage. d. increased jugular venous distention.

ANS: D Increases in jugular venous distention in a patient with a subarachnoid hemorrhage may indicate an increase in intracranial pressure (ICP) and that the PEEP setting is too high for this patient. A respiratory rate of 20, O2 saturation of 93%, and green nasogastric tube drainage are within normal limits. DIF: Cognitive Level: Apply (application) REF: 1623-1624 TOP: Nursing Process: Assessment MSC: NCLEX: Physiological Integrity

7. When monitoring the effectiveness of treatment for a patient with a large anterior wall myocardial infarction, the most pertinent measurement for the nurse to obtain is a. central venous pressure (CVP). b. systemic vascular resistance (SVR). c. pulmonary vascular resistance (PVR). d. pulmonary artery wedge pressure (PAWP).

ANS: D PAWP reflects left ventricular end diastolic pressure (or left ventricular preload) and is a sensitive indicator of cardiac function. Because the patient is high risk for left ventricular failure, the PAWP must be monitored. An increase will indicate left ventricular failure. The other values would also provide useful information, but the most definitive measurement of changes in cardiac function is the PAWP.

7. When monitoring for the effectiveness of treatment for a patient with a large anterior wall myocardial infarction, the most important information for the nurse to obtain is a. central venous pressure (CVP). b. systemic vascular resistance (SVR). c. pulmonary vascular resistance (PVR). d. pulmonary artery wedge pressure (PAWP).

ANS: D PAWP reflects left ventricular end diastolic pressure (or left ventricular preload) and is a sensitive indicator of cardiac function. Because the patient is high risk for left ventricular failure, the PAWP must be monitored. An increase will indicate left ventricular failure. The other values would also provide useful information, but the most definitive measurement of changes in cardiac function is the PAWP. DIF: Cognitive Level: Apply (application) REF: 1607 TOP: Nursing Process: Evaluation MSC: NCLEX: Physiological Integrity

37. After change-of-shift report on a ventilator weaning unit, which patient should the nurse assess first? a. Patient who failed a spontaneous breathing trial and has been placed in a rest mode on the ventilator b. Patient who is intubated and has continuous partial pressure end-tidal CO2 (PETCO2) monitoring c. Patient with a central venous oxygen saturation (ScvO2) of 69% while on bilevel positive airway pressure (BiPAP) d. Patient who was successfully weaned and extubated 4 hours ago and now has no urine output for the last 6 hours

ANS: D The decreased urine output may indicate acute kidney injury or that the patient's cardiac output and perfusion of vital organs have decreased. Any of these causes would require rapid action. The data about the other patients indicate that their conditions are stable and do not require immediate assessment or changes in their care. Continuous PETCO2 monitoring is frequently used when patients are intubated. The rest mode should be used to allow patient recovery after a failed SBT, and an ScvO2 of 69% is within normal limits. DIF: Cognitive Level: Analyze (analysis) REF: 1625 | 1627 OBJ: Special Questions: Prioritization; Multiple Patients TOP: Nursing Process: Planning MSC: NCLEX: Safe and Effective Care Environment

37. After change-of-shift report, which patient should the progressive care nurse assess first? a. Patient who was extubated this morning and has a temperature of 101.4°F (38.6°C) b. Patient with bilevel positive airway pressure (BiPAP) for obstructive sleep apnea and a respiratory rate of 16 c. Patient with arterial pressure monitoring who is 2 hours post-percutaneous coronary intervention and needs to void d. Patient who is receiving IV heparin for a venous thromboembolism and has a partial thromboplastin time (PTT) of 101 sec

ANS: D The findings for this patient indicate high risk for bleeding from an elevated (nontherapeutic) PTT. The nurse needs to adjust the rate of the infusion (dose) per the health care provider's parameters. The patient with BiPAP for sleep apnea has a normal respiratory rate. The patient recovering from the percutaneous coronary intervention will need to be assisted with voiding and this task could be delegated to unlicensed assistive personnel. The patient with a fever may be developing ventilator-associated pneumonia, but addressing the bleeding risk is a higher priority.

38. After change-of-shift report, which patient should the progressive care nurse assess first? a. Patient who was extubated in the morning and has a temperature of 101.4° F (38.6° C) b. Patient with bilevel positive airway pressure (BiPAP) for sleep apnea whose respiratory rate is 16 c. Patient with arterial pressure monitoring who is 2 hours post-percutaneous coronary intervention who needs to void d. Patient who is receiving IV heparin for a venous thromboembolism and has a partial thromboplastin time (PTT) of 98 sec

ANS: D The findings for this patient indicate high risk for bleeding from an elevated (nontherapeutic) PTT. The nurse needs to adjust the rate of the infusion (dose) per the health care provider's parameters. The patient with BiPAP for sleep apnea has a normal respiratory rate. The patient recovering from the percutaneous coronary intervention will need to be assisted with voiding and this task could be delegated to unlicensed assistive personnel. The patient with a fever may be developing ventilator-associated pneumonia, but addressing the bleeding risk is a higher priority. DIF: Cognitive Level: Analyze (analysis) REF: 1600 OBJ: Special Questions: Prioritization; Multiple Patients TOP: Nursing Process: Planning MSC: NCLEX: Safe and Effective Care Environment

31. The nurse educator is evaluating the care that a new registered nurse (RN) provides to a patient receiving mechanical ventilation. Which action by the new RN indicates the need for more education? a. The RN increases the FIO2 to 100% before suctioning. b. The RN secures a bite block in place using adhesive tape. c. The RN asks for assistance to resecure the endotracheal tube. d. The RN positions the patient with the head of bed at 10 degrees.

ANS: D The head of the patient's bed should be positioned at 30 to 45 degrees to prevent ventilator-associated pneumonia. The other actions by the new RN are appropriate.

15. While waiting for cardiac transplantation, a patient with severe cardiomyopathy has a ventricular assist device (VAD) implanted. When planning care for this patient, the nurse should anticipate a. giving immunosuppressive medications. b. preparing the patient for a permanent VAD. c. teaching the patient the reason for complete bed rest. d. monitoring the surgical incision for signs of infection.

ANS: D The insertion site for the VAD provides a source for transmission of infection to the circulatory system and requires frequent monitoring. Patient's with VADs are able to have some mobility and may not be on bed rest. The VAD is a bridge to transplantation, not a permanent device. Immunosuppression is not necessary for nonbiologic devices like the VAD. DIF: Cognitive Level: Apply (application) REF: 1613 TOP: Nursing Process: Planning MSC: NCLEX: Physiological Integrity

17. To maintain proper cuff pressure of an endotracheal tube (ET) when the patient is on mechanical ventilation, the nurse should a. inflate the cuff with a minimum of 10 mL of air. b. inflate the cuff until the pilot balloon is firm on palpation. c. inject air into the cuff until a manometer shows 15 mm Hg pressure. d. inject air into the cuff until a slight leak is heard only at peak inflation.

ANS: D The minimal occluding volume technique involves injecting air into the cuff until an air leak is present only at peak inflation. The volume to inflate the cuff varies with the ET and the patient's size. Cuff pressure should be maintained at 20 to 25 mm Hg. An accurate assessment of cuff pressure cannot be obtained by palpating the pilot balloon.

17. To maintain proper cuff pressure of an endotracheal tube (ET) when the patient is on mechanical ventilation, the nurse should a. inflate the cuff with a minimum of 10 mL of air. b. inflate the cuff until the pilot balloon is firm on palpation. c. inject air into the cuff until a manometer shows 15 mm Hg pressure. d. inject air into the cuff until a slight leak is heard only at peak inflation.

ANS: D The minimal occluding volume technique involves injecting air into the cuff until an air leak is present only at peak inflation. The volume to inflate the cuff varies with the ET and the patient's size. Cuff pressure should be maintained at 20 to 25 mm Hg. An accurate assessment of cuff pressure cannot be obtained by palpating the pilot balloon. DIF: Cognitive Level: Understand (comprehension) REF: 1615 TOP: Nursing Process: Implementation MSC: NCLEX: Physiological Integrity

30. The nurse responds to a ventilator alarm and finds the patient lying in bed holding the endotracheal tube (ET). Which action should the nurse take next? a. Activate the rapid response team. b. Provide reassurance to the patient. c. Call the health care provider to reinsert the tube. d. Manually ventilate the patient with 100% oxygen.

ANS: D The nurse should ensure maximal patient oxygenation by manually ventilating with a bag-valve-mask system. Offering reassurance to the patient, notifying the health care provider about the need to reinsert the tube, and activating the rapid response team are also appropriate after the nurse has stabilized the patient's oxygenation. DIF: Cognitive Level: Apply (application) REF: 1617 OBJ: Special Questions: Prioritization TOP: Nursing Process: Implementation MSC: NCLEX: Physiological Integrity

21. Four hours after mechanical ventilation is initiated, a patient's arterial blood gas (ABG) results include a pH of 7.51, PaO2 of 82 mm Hg, PaCO2 of 26 mm Hg, and HCO3 - of 23 mEq/L (23 mmol/L). The nurse will anticipate the need to a. increase the FIO2 . c. increase the respiratory rate. b. increase the tidal volume. d. decrease the respiratory rate.

ANS: D The patient's PaCO2 and pH indicate respiratory alkalosis caused by too high a respiratory rate. The PaO2 is appropriate for a patient with COPD and increasing the respiratory rate and tidal volume would further lower the PaCO2 .

21. Four hours after mechanical ventilation is initiated for a patient with chronic obstructive pulmonary disease (COPD), the patient's arterial blood gas (ABG) results include a pH of 7.51, PaO2 of 82 mm Hg, PaCO2 of 26 mm Hg, and HCO3- of 23 mEq/L (23 mmol/L). The nurse will anticipate the need to a. increase the FIO2. b. increase the tidal volume. c. increase the respiratory rate. d. decrease the respiratory rate.

ANS: D The patient's PaCO2 and pH indicate respiratory alkalosis caused by too high a respiratory rate. The PaO2 is appropriate for a patient with COPD and increasing the respiratory rate and tidal volume would further lower the PaCO2. DIF: Cognitive Level: Analyze (analysis) REF: 1615-1616 TOP: Nursing Process: Planning MSC: NCLEX: Physiological Integrity

10. While assisting with the placement of a pulmonary artery (PA) catheter, the nurse notes that the catheter is correctly placed when the balloon is inflated and the monitor shows a a. typical PA pressure waveform. b. tracing of the systemic arterial pressure. c. tracing of the systemic vascular resistance. d. typical PA wedge pressure (PAWP) tracing.

ANS: D The purpose of a PA line is to measure PAWP, so the catheter is floated through the pulmonary artery until the dilated balloon wedges in a distal branch of the pulmonary artery, and the PAWP readings are available. After insertion, the balloon is deflated and the PA waveform will be observed. Systemic arterial pressures are obtained using an arterial line, and the systemic vascular resistance is a calculated value, not a waveform.

10. When assisting with the placement of a pulmonary artery (PA) catheter, the nurse notes that the catheter is correctly placed when the monitor shows a a. typical PA pressure waveform. b. tracing of the systemic arterial pressure. c. tracing of the systemic vascular resistance. d. typical PA wedge pressure (PAWP) tracing.

ANS: D The purpose of a PA line is to measure PAWP, so the catheter is floated through the pulmonary artery until the dilated balloon wedges in a distal branch of the pulmonary artery, and the PAWP readings are available. After insertion, the balloon is deflated and the PA waveform will be observed. Systemic arterial pressures are obtained using an arterial line and the systemic vascular resistance is a calculated value, not a waveform. DIF: Cognitive Level: Understand (comprehension) REF: 1608 TOP: Nursing Process: Assessment MSC: NCLEX: Physiological Integrity

25. When caring for the patient with a pulmonary artery (PA) pressure catheter, the nurse observes that the PA waveform indicates that the catheter is in the wedged position. Which action should the nurse take next? a. Zero balance the transducer. b. Activate the fast flush system. c. Notify the health care provider. d. Deflate and reinflate the PA balloon.

ANS: D When the catheter is in the wedge position, blood flow past the catheter is obstructed, placing the patient at risk for pulmonary infarction. A health care provider or advanced practice nurse should be called to reposition the catheter. The other actions will not correct the wedging of the PA catheter. DIF: Cognitive Level: Apply (application) REF: 1608 TOP: Nursing Process: Implementation MSC: NCLEX: Physiological Integrity

2 To ensure adequate tracheal perfusion, the nurse should maintain cuff pressure at 20 to 25 cm H2O. Excess cuff pressure can damage the tracheal mucosa. Lesser cuff pressure may cause the ET tube to become destabilized and extubate. Text Reference - p. 1615

An endotracheal (ET) tube is inserted in a patient. The nurse inflates the cuff to stabilize the tube. How much cuff pressure should be maintained to keep it inflated and ensure adequate tracheal perfusion? 1 10-15 cm H2O 2 20-25 cm H2O 3 30-35 cm H2O 4 40-45 cm H2O

1, 3, 4 Whenever a patient is intubated, repositioning and retaping the tube after 24 hours are essential. For the orally intubated patient, remove the bite block and the old tape or ties. Provide oral hygiene, and then reposition the ET tube to the opposite side of the mouth. There is no need to give anesthesia for maintaining oral care. There is no point in physically restraining the patient, because it will cause discomfort and anxiety. Text Reference - p. 1617

An endotracheal (ET) tube is placed for a patient. What interventions should the nurse perform to maintain oral hygiene and care? Select all that apply. 1 Reposition and retape the ET tube at least every 24 hours. 2 Keep the buccal cavity and nasal cavity anesthetized. 3 Replace the bite block and reconfirm proper cuff inflation and tube placement regularly. 4 Provide oral hygiene with repositioning of the ET tube to the opposite side of the mouth after care. 5 Keep the patient restrained to avoid dislodging the tube.

When planning care for a patient on a mechanical ventilator, the nurse understands that the application of positive end-expiratory pressure (PEEP) to the ventilator settings has which therapeutic effect? A. Increased inflation of the lungs B. Prevention of barotrauma to the lung tissue C. Prevention of alveolar collapse during expiration D. Increased fraction of inspired oxygen concentration (FIO2) administration

Ans. C PEEP is positive pressure that is applied to the airway during exhalation. This positive pressure prevents the alveoli from collapsing, improving oxygenation and enabling a reduced FIO2 requirement. PEEP does not cause increased inflation of the lungs or prevent barotrauma. Actually auto-PEEP resulting from inadequate exhalation time may contribute to barotrauma.

The patient has developed cardiogenic shock after a left anterior descending myocardial infection. Which circulatory-assist device should the nurse expect to use for this patient? A. Cardiopulmonary bypass B. Impedance cardiography (ICG) C. Intraaortic balloon pump (IABP) D. Central venous pressure (CVP) measurement

Ans. C The most commonly used mechanical circulatory-assist device is the intraaortic balloon pump (IABP), and it is used to decrease ventricular workload, increase myocardial perfusion, and augment circulation. Cardiopulmonary bypass provides circulation during open heart surgery. It is not used as an assist device after surgery. ICG is a noninvasive method to obtain cardiac output and assess thoracic fluid status. CVP measurement is an invasive measurement of right ventricular preload and reflects fluid volume problems

The post-anesthesia care unit (PACU) has several patients with endotracheal tubes. Which patient should receive the least amount of endotracheal suctioning? A. Transplantation of a kidney B. Replacement of aortic valve C. Cerebral aneurysm resection D. Formation of an ileal conduit

Ans. C The nurse should avoid suctioning the patient after a craniotomy until it is necessary because suctioning will increase this patient's intracranial pressure. The patients with a kidney transplantation, aortic valve replacement, or formation of an ileal conduit will not be negatively affected by suctioning, although it should only be done when needed, not routinely.

Which interventions should the nurse perform prior to suctioning a patient who has an endotracheal (ET) tube using open-suction technique (select all that apply)? A. Put on clean gloves. B. Administer a bronchodilator. C. Perform a cardiopulmonary assessment. D. Hyperoxygenate the patient for 30 seconds. E. Insert a few drops of normal saline into the ET to break up secretions

Ans. C, D Suctioning is preceded by a thorough assessment and hyperoxygenation for 30 seconds. Sterile, not clean, gloves are necessary, and it is not necessary to administer a bronchodilator. Instillation of normal saline into the ET tube is not an accepted standard practice

Which hematologic problem most significantly increases the risks associated with pulmonary artery (PA) catheter insertion? A. Leukocytosis B. Hypovolemia C. Hemolytic anemia D. Thrombocytopenia

Ans. D PA catheter insertion carries a significant risk of bleeding, a fact that is exacerbated when the patient has low levels of platelets. Leukocytosis, hypovolemia, and anemia are less likely to directly increase the risks associated with PA insertion

A patient's daughter asks the nurse what SIMV means on the settings of the mechanical ventilator attached to her father. Which statement best describes this mode of ventilation? A. "SIMV provides additional inspiratory pressure so that your father does not have to work as hard to breathe, thus enabling better oxygenation and a quicker recovery with fewer complications." B. "SIMV is a mode that allows the ventilator to totally control your father's breathing. It will prevent him from hyperventilating or hypoventilating, thus ensuring the best oxygenation." C. "SIMV is a mode that allows your father to breathe on his own, but the ventilator will control how deep a breath he will receive. The ventilator can sense when he wants a breath, and it will deliver it." D. "SIMV is a mode that allows your father to breathe on his own while receiving a preset number of breaths from the ventilator. He can breathe as much or as little as he wants beyond what the ventilator will breathe for him."

Ans. D SIMV stands for synchronized intermittent mandatory ventilation, a mode of ventilation in which the ventilator delivers a preset tidal volume at a preset frequency in synchrony with the patient's spontaneous breathing. Between ventilator-delivered breaths the patient is able to breathe spontaneously, receiving the preset FIO2 but self-regulates the rate and depth of those breaths. Pressure support ventilation (PSV) applies positive pressure only during inspiration. PSV is not used as a sole ventilator support during acute respiratory failure because of the risk of hypoventilation, but it does decrease the work of breathing. Pressure-control inverse ratio ventilation (PC-IRV) sets the ventilation pressure and the ratio of inspiration to expiration to control the patient's breathing. Assist-control ventilation (ACV) or assisted mandatory ventilation (AMV) delivers a preset rate of breaths but allows the patient to breathe spontaneously, with a preset tidal volume

The nurse is caring for a patient who has been on a mechanical ventilator for several days. Which weaning parameter would tell the nurse whether or not the patient has enough muscle strength to breathe without assistance? A. Tidal volume B. Minute ventilation C. Forced vital capacity D. Negative inspiratory force

Ans. D The negative inspiratory force (NIF) measures inspiratory muscle strength. Tidal volume and minute ventilation assess the patient's respiratory endurance. Forced vital capacity is not used as a measure to determine weaning from a ventilator

A patient is scheduled for a myelogram and the nurse explains to the patient that this is an invasive procedure, which assesses for any lesions in the spinal cord. The nurse should explain that the preparation is similar to which of the following neurologic tests? A) Lumbar puncture B) MRI C) Cerebral angiography D) EEG

Ans: A Feedback: A myelogram is an x-ray of the spinal subarachnoid space taken after the injection of a contrast agent into the spinal subarachnoid space through a lumbar puncture. Patient preparation for a myelogram would be similar to that for lumbar puncture. The other listed diagnostic tests do not involve lumbar puncture.

A patient with lower back pain is scheduled for myelography using metrizamide (a water-soluble contrast dye). After the test, the nurse should prioritize what action? A) Positioning the patient with the head of the bed elevated 45 degrees B) Administering IV morphine sulfate to prevent headache C) Limiting fluids for the next 12 hours D) Helping the patient perform deep breathing and coughing exercises

Ans: A Feedback: After myelography, the patient lies in bed with the head of the bed elevated 30 to 45 degrees. The patient is advised to remain in bed in the recommended position for 3 hours or as prescribed. Drinking liberal amounts of fluid for rehydration and replacement of CSF may decrease the incidence of post-lumbar puncture headache. Deep breathing and coughing exercises are not normally necessary since there is no consequent risk of atelectasis.

The nurse is doing an initial assessment on a patient newly admitted to the unit with a diagnosis of cerebrovascular accident (CVA). The patient has difficulty copying a figure that the nurse has drawn and is diagnosed with visual-receptive aphasia. What brain region is primarily involved in this deficit? A) Temporal lobe B) Parietal-occipital area C) Inferior posterior frontal areas D) Posterior frontal area

Ans: B Difficulty copying a figure that the nurse has drawn would be considered visual-receptive aphasia, which involves the parietal-occipital area. Expressive aphasia, the inability to express oneself, is often associated with damage to the frontal area. Receptive aphasia, the inability to understand what someone else is saying, is often associated with damage to the temporal lobe area.

The nurse is caring for a patient who exhibits abnormal results of the Weber test and Rinne test. The nurse should suspect dysfunction involving what cranial nerve? A) Trigeminal B) Acoustic C) Hypoglossal D) Trochlear

Ans: B Feedback: Abnormal hearing can correlate with damage to cranial nerve VIII (acoustic). The acoustic nerve functions in hearing and equilibrium. The trigeminal nerve functions in facial sensation, corneal reflex, and chewing. The hypoglossal nerve moves the tongue. The trochlear nerve controls muscles that move the eye.

The nurse is performing a neurologic assessment of a patient whose injuries have rendered her unable to follow verbal commands. How should the nurse proceed with assessing the patient's level of consciousness (LOC)? A) Assess the patient's vital signs and correlate these with the patient's baselines. B) Assess the patient's eye opening and response to stimuli. C) Document that the patient currently lacks a level of consciousness. D) Facilitate diagnostic testing in an effort to obtain objective data.

Ans: B Feedback: If the patient is not alert or able to follow commands, the examiner observes for eye opening; verbal response and motor response to stimuli, if any; and the type of stimuli needed to obtain a response. Vital signs and diagnostic testing are appropriate, but neither will allow the nurse to gauge the patient's LOC. Inability to follow commands does not necessarily denote an absolute lack of consciousness.

In the course of a focused neurologic assessment, the nurse is palpating the patient's major muscle groups at rest and during passive movement. Data gleaned from this assessment will allow the nurse to describe which of the following aspects of neurologic function? A) Muscle dexterity B) Muscle tone C) Motor symmetry D) Deep tendon reflexes

Ans: B Feedback: Muscle tone (the tension present in a muscle at rest) is evaluated by palpating various muscle groups at rest and during passive movement. Data from this assessment do not allow the nurse to ascertain the patient's dexterity, reflexes, or motor symmetry

The nurse is planning the care of a patient with Parkinson's disease. The nurse should be aware that treatment will focus on what pathophysiological phenomenon? A) Premature degradation of acetylcholine B) Decreased availability of dopamine C) Insufficient synthesis of epinephrine D) Delayed reuptake of serotonin

Ans: B Feedback: Parkinson's disease develops from decreased availability of dopamine, not acetylcholine, epinephrine, or serotonin.

The physician has ordered a somatosensory evoked responses (SERs) test for a patient for whom the nurse is caring. The nurse is justified in suspecting that this patient may have a history of what type of neurologic disorder? A) Hypothalamic disorder B) Demyelinating disease C) Brainstem deficit D) Diabetic neuropathy

Ans: B Feedback: SERs are used to detect deficits in the spinal cord or peripheral nerve conduction and to monitor spinal cord function during surgical procedures. The test is also useful in the diagnosis of demyelinating diseases, such as multiple sclerosis and polyneuropathies, where nerve conduction is slowed. The test is not done to diagnose hypothalamic disorders, brainstem deficits, or diabetic neuropathies.

A trauma patient in the ICU has been declared brain dead. What diagnostic test is used in making the determination of brain death? A) Magnetic resonance imaging (MRI) B) Electroencephalography (EEG) C) Electromyelography (EMG) D) Computed tomography (CT

Ans: B Feedback: The EEG can be used in determining brain death. MRI, CT, and EMG are not normally used in determining brain death.

Assessment is crucial to the care of patients with neurologic dysfunction. What does accurate and appropriate assessment require? Select all that apply. A) The ability to select mediations for the neurologic dysfunction B) Understanding of the tests used to diagnose neurologic disorders C) Knowledge of nursing interventions related to assessment and diagnostic testing D) Knowledge of the anatomy of the nervous system E) The ability to interpret the results of diagnostic tests

Ans: B, C, D Feedback: Assessment requires knowledge of the anatomy and physiology of the nervous system and an understanding of the array of tests and procedures used to diagnose neurologic disorders. Knowledge about the nursing implications and interventions related to assessment and diagnostic testing is also essential. Selecting medications and interpreting diagnostic tests are beyond the normal scope of the nurse.

The nurse educator is reviewing the assessment of cranial nerves. What should the educator identify as the specific instances when cranial nerves should be assessed? Select all that apply. A) When a neurogenic bladder develops B) When level of consciousness is decreased C) With brain stem pathology D) In the presence of peripheral nervous system disease E) When a spinal reflex is interrupted

Ans: B, C, D Feedback: Cranial nerves are assessed when level of consciousness is decreased, with brain stem pathology, or in the presence of peripheral nervous system disease. Abnormalities in muscle tone and involuntary movements are less likely to prompt the assessment of cranial nerves, since these nerves do not directly mediate most aspects of muscle tone and movement.

A patient had a lumbar puncture performed at the outpatient clinic and the nurse has phoned the patient and family that evening. What does this phone call enable the nurse to determine? A) What are the patient's and family's expectations of the test B) Whether the patient's family had any questions about why the test was necessary C) Whether the patient has had any complications of the test D) Whether the patient understood accurately why the test was done

Ans: C Feedback: Contacting the patient and family after diagnostic testing enables the nurse to determine whether they have any questions about the procedure or whether the patient had any untoward results. The other listed information should have been elicited from the patient and family prior to the test.

When caring for a patient with an altered level of consciousness, the nurse is preparing to test cranial nerve VII. What assessment technique would the nurse use to elicit a response from cranial nerve VII? A) Palpate trapezius muscle while patient shrugs should against resistance. B) Administer the whisper or watch-tick test. C) Observe for facial movement symmetry, such as a smile. D) Note any hoarseness in the patient's voice.

Ans: C Feedback: Cranial nerve VII is the facial nerve. An appropriate assessment technique for this cranial nerve would include observing for symmetry while the patient performs facial movements: smiles, whistles, elevates eyebrows, and frowns. Palpating and noting strength of the trapezius muscle while the patient shrugs shoulders against resistance would be completed to assess cranial nerve XI (spinal accessory). Assessing cranial nerve VIII (acoustic) would involve using the whisper or watch-tick test to evaluate hearing. Noting any hoarseness in the patient's voice would involve assessment of cranial nerve X (vagus)

A patient is scheduled for CT scanning of the head because of a recent onset of neurologic deficits. What should the nurse tell the patient in preparation for this test? A) "No metal objects can enter the procedure room." B) "You need to fast for 8 hours prior to the test." C) "You will need to lie still throughout the procedure." D) "There will be a lot of noise during the test."

Ans: C Feedback: Preparation for CT scanning includes teaching the patient about the need to lie quietly throughout the procedure. If the patient were having an MRI, metal and noise would be appropriate teaching topics. There is no need to fast prior to a CT scan of the brain.

During the performance of the Romberg test, the nurse observes that the patient sways slightly. What is the nurse's most appropriate action? A) Facilitate a referral to a neurologist. B) Reposition the patient supine to ensure safety. C) Document successful completion of the assessment. D) Follow up by having the patient perform the Rinne test.

Ans: C Feedback: Slight swaying during the Romberg test is normal, but a loss of balance is abnormal and is considered a positive Romberg test. Slight swaying is not a significant threat to the patient's safety. The Rinne test assesses hearing, not balance.

A patient is having a "fight or flight response" after receiving bad news about his prognosis. What affect will this have on the patient's sympathetic nervous system? A) Constriction of blood vessels in the heart muscle B) Constriction of bronchioles C) Increase in the secretion of sweat D) Constriction of pupils

Ans: C Feedback: Sympathetic nervous system stimulation results in dilated blood vessels in the heart and skeletal muscle, dilated bronchioles, increased secretion of sweat, and dilated pupils.

A trauma patient was admitted to the ICU with a brain injury. The patient had a change in level of consciousness, increased vital signs, and became diaphoretic and agitated. The nurse should recognize which of the following syndromes as the most plausible cause of these symptoms? A) Adrenal crisis B) Hypothalamic collapse C) Sympathetic storm D) Cranial nerve deficit

Ans: C Feedback: Sympathetic storm is a syndrome associated with changes in level of consciousness, altered vital signs, diaphoresis, and agitation that may result from hypothalamic stimulation of the sympathetic nervous system following traumatic brain injury. Alterations in cranial nerve or adrenal function would not have this result.

The nurse caring for an 80 year-old patient knows that she has a pre-existing history of dulled tactile sensation. The nurse should first consider what possible cause for this patient's diminished tactile sensation? A) Damage to cranial nerve VIII B) Adverse medication effects C) Age-related neurologic changes D) An undiagnosed cerebrovascular accident in early adulthood

Ans: C Feedback: Tactile sensation is dulled in the elderly person due to a decrease in the number of sensory receptors. While thorough assessment is necessary, it is possible that this change is unrelated to pathophysiological processes.

A patient in the OR goes into malignant hyperthermia due to an abnormal reaction to the anesthetic. The nurse knows that the area of the brain that regulates body temperature is which of the following? A) Cerebellum B) Thalamus C) Hypothalamus D) Midbrain

Ans: C Feedback: The hypothalamus plays an important role in the endocrine system because it regulates the pituitary secretion of hormones that influence metabolism, reproduction, stress response, and urine production. It works with the pituitary to maintain fluid balance through hormonal release and maintains temperature regulation by promoting vasoconstriction or vasodilatation. The cerebellum, thalamus, and midbrain and not directly involved in temperature regulation.

The nurse is caring for a patient with an upper motor neuron lesion. What clinical manifestations should the nurse anticipate when planning the patient's neurologic assessment? A) Decreased muscle tone B) Flaccid paralysis C) Loss of voluntary control of movement D) Slow reflexes

Ans: C Upper motor neuron lesions do not cause muscle atrophy, flaccid paralysis, or slow reflexes. However, upper motor neuron lesions normally cause loss of voluntary control.

A 72-year-old man has been brought to his primary care provider by his daughter, who claims that he has been experiencing uncharacteristic lapses in memory. What principle should underlie the nurse's assessment and management of this patient? A) Loss of short-term memory is normal in older adults, but loss of long-term memory is pathologic. B) Lapses in memory in older adults are considered benign unless they have negative consequences. C) Gradual increases in confusion accompany the aging process. D) Thorough assessment is necessary because changes in cognition are always considered to be pathologic

Ans: D Feedback: Although mental processing time decreases with age, memory, language, and judgment capacities remain intact. Change in mental status should never be assumed to be a normal part of aging.

The neurologic nurse is testing the function of a patient's cerebellum and basal ganglia. What action will most accurately test these structures? A) Have the patient identify the location of a cotton swab on his or her skin with the eyes closed. B) Elicit the patient's response to a hypothetical problem. C) Ask the patient to close his or her eyes and discern between hot and cold stimuli. D) Guide the patient through the performance of rapid, alternating movements.

Ans: D Feedback: Cerebellar and basal ganglia influence on the motor system is reflected in balance control and coordination. Coordination in the hands and upper extremities is tested by having the patient perform rapid, alternating movements and point-to-point testing. The cerebellum and basal ganglia do not mediate cutaneous sensation or judgment.

The nursing students are learning how to assess function of cranial nerve VIII. To assess the function of cranial nerve VIII the students would be correct in completing which of the following assessment techniques? A) Have the patient identify familiar odors with the eyes closed. B) Assess papillary reflex. C) Utilize the Snellen chart. D) Test for air and bone conduction (Rinne test).

Ans: D Feedback: Cranial nerve VIII is the acoustic nerve. It functions in hearing and equilibrium. When assessing this nerve, the nurse would test for air and bone conduction (Rinne) with a tuning fork. Assessment of papillary reflex would be completed for cranial nerves III (oculomotor), IV (trochlear), and VI (abducens). The Snellen chart would be used to assess cranial nerve II (optic).

A patient for whom the nurse is caring has positron emission tomography (PET) scheduled. In preparation, what should the nurse explain to the patient? A) The test will temporarily limit blood flow through the brain. B) An allergy to iodine precludes getting the radio-opaque dye. C) The patient will need to endure loud noises during the test. D) The test may result in dizziness or lightheadedness.

Ans: D Feedback: Key nursing interventions for PET scan include explaining the test and teaching the patient about inhalation techniques and the sensations (e.g., dizziness, light-headedness, and headache) that may occur. A PET scan does not impede blood flow through the brain. An allergy to iodine precludes the dye for an MRI, and loud noise is heard in an MRI.

The nurse is conducting a focused neurologic assessment. When assessing the patient's cranial nerve function, the nurse would include which of the following assessments? A) Assessment of hand grip B) Assessment of orientation to person, time, and place C) Assessment of arm drift D) Assessment of gag reflex

Ans: D Feedback: The gag reflex is governed by the glossopharyngeal nerve, one of the cranial nerves. Hand grip and arm drifting are part of motor function assessment. Orientation is an assessment parameter related to a mental status examination.

A patient is currently being stimulated by the parasympathetic nervous system. What effect will this nervous stimulation have on the patient's bladder? A) The parasympathetic nervous system causes urinary retention. B) The parasympathetic nervous system causes bladder spasms. C) The parasympathetic nervous system causes urge incontinence. D) The parasympathetic nervous system makes the bladder contract.

Ans: D Feedback: The parasympathetic division of the nervous system causes contraction (stimulation) of the urinary bladder muscles and a decrease (inhibition) in heart rate, whereas the sympathetic division produces relaxation (inhibition) of the urinary bladder and an increase (stimulation) in the rate and force of the heartbeat.

The nurse is admitting a patient to the unit who is diagnosed with a lower motor neuron lesion. What entry in the patient's electronic record is most consistent with this diagnosis? A) "Patient exhibits increased muscle tone." B) "Patient demonstrates normal muscle structure with no evidence of atrophy." C) "Patient demonstrates hyperactive deep tendon reflexes." D) "Patient demonstrates an absence of deep tendon reflexes."

Ans: D Lower motor neuron lesions cause flaccid muscle paralysis, muscle atrophy, decreased muscle tone, and loss of voluntary control.

An elderly patient is being discharged home. The patient lives alone and has atrophy of his olfactory organs. The nurse tells the patient's family that it is essential that the patient have what installed in the home? A) Grab bars B) Nonslip mats C) Baseboard heaters D) A smoke detector

Ans: D The sense of smell deteriorates with age. The olfactory organs are responsible for smell. This may present a safety hazard for the patient because he or she may not smell smoke or gas leaks. Smoke detectors are universally necessary, but especially for this patient.

What term is used to describe the fibrous connective tissue that hugs the brain closely and extends into every fold of the brain's surface? A) Dura mater B) Arachnoid C) Fascia D) Pia mater

Ans: D The term "meninges" describes the fibrous connective tissue that covers the brain and spinal cord. The meninges have three layers, the dura mater, arachnoid, and pia mater. The pia mater is the innermost membrane that hugs the brain closely and extends into every fold of the brain's surface. The dura mater, the outermost layer, covers the brain and spinal cord. The arachnoid, the middle membrane, is responsible for the production of cerebrospinal fluid.

3 Auto-positive end-expiratory pressure (auto-PEEP) is caused by inadequate exhalation time. Barotrauma, hemodynamic instability and increased work of breathing are the results and not the causes of the auto-PEEP. Text Reference - p. 1621

Auto-positive end-expiratory pressure (PEEP) is the additional PEEP over what is set by the health care provider. What causes auto-PEEP during mechanical ventilation? 1 Barotrauma 2 Hemodynamic instability 3 Inadequate exhalation time 4 Increased work of breathing

A nurse is assessing reflexes in a patient with hyperactive reflexes. When the patient's foot is abruptly dorsiflexed, it continues to "beat" two to three times before settling into a resting position. How would the nurse document this finding? A) Rigidity B) Flaccidity C) Clonus D) Ataxia

C When reflexes are very hyperactive, a phenomenon called clonus may be elicited. If the foot is abruptly dorsiflexed, it may continue to "beat" two to three times before it settles into a position of rest. Rigidity is an increase in muscle tone at rest characterized by increased resistance to passive stretch. Flaccidity is lack of muscle tone. Ataxia is the inability to coordinate muscle movements, resulting in difficulty walking, talking, and performing self-care activities.

A 68-yr-old male patient diagnosed with sepsis is orally intubated on mechanical ventilation. Which nursing action is most important? Use the open-suctioning technique. Administer morphine for discomfort. Limit noise and cluster care activities. Elevate the head of the bed 30 degrees.

Elevate the head of the bed 30 degrees. The two major complications of endotracheal intubation are unplanned extubation and aspiration. To prevent aspiration, all intubated patients and patients receiving enteral feedings must have the head of the bed elevated a minimum of 30 to 45 degrees unless medically contraindicated. Closed-suction technique is preferred over the open-suction technique because oxygenation and ventilation are maintained during suctioning, and exposure to secretions is reduced. The nurse should provide comfort measures such as morphine to relieve anxiety and pain associated with intubation. To promote rest and sleep, the nurse should limit noise and cluster activities.

The nurse is caring for a 65-yr-old man with acute respiratory distress syndrome (ARDS) who is on pressure support ventilation (PSV), fraction of inspired oxygen (FIO2) at 80%, and positive end-expiratory pressure (PEEP) at 15 cm H2O. The patient weighs 72 kg. What finding would indicate that treatment is effective? PaO2 of 60 mm Hg Tidal volume of 700 mL Cardiac output of 2.7 L/min Inspiration to expiration ratio of 1:2

PaO2 of 60 mm Hg Severe hypoxemia (PaO2 less than 40 mm Hg) occurs with ARDS, and PEEP is increased to improve oxygenation and prevent oxygen toxicity by reducing FIO2. A PaO2 level of 60 mm Hg indicates that treatment is effective and oxygenation status has improved. Decreased cardiac output is a complication of PEEP. Normal cardiac output is 4 to 8 L/minute. Normal tidal volume is 6 to 10 mL/kg. PSV delivers a preset pressure but the tidal volume varies with each breath. I:E ratio is usually set at 1:2 to 1:1.5 and does not indicate patient improvement.

Which interventions should the nurse perform before using an open-suctioning technique for a patient with an endotracheal (ET) tube (select all that apply.)? Put on clean gloves. Administer a bronchodilator. Perform a cardiopulmonary assessment. Hyperoxygenate the patient for 30 seconds. Perform hand hygiene before performing the procedure. Insert a few drops of normal saline into the ET to break up secretions.

Perform a cardiopulmonary assessment. Hyperoxygenate the patient for 30 seconds. Suctioning is preceded by a thorough assessment and hyperoxygenation for 30 seconds. Sterile, not clean, gloves are necessary, and it is not necessary to administer a bronchodilator. Instillation of normal saline into the ET tube is not an accepted standard practice.

3 Certification as an advance care nurse practitioner (ACNP) is available through the American Association of Critical Care Nurses (AACN). The National League of Nurses (NLN), the American Association of Nurse Practitioners (AANP), and the National Council of State Boards of Nursing (NCSBN) are all nursing organizations but they do not offer certification to the advance practice registered nurse (APRN) as an ACNP. Text Reference - p. 1600

The advance practice registered nurse (APRN) is seeking certification as an advance care nurse practitioner (ACNP). Which professional organization can grant this certification to the APRN? 1 National League of Nurses (NLN) 2 American Association of Nurse Practitioners (AANP) 3 American Association of Critical Care Nurses (AACN) 4 National Council of State Boards of Nursing (NCSBN

1, 3, 4 In order to achieve the designation as a pediatric critical care nurse (CCRN) the nurse will require registered nurse licensure, successful completion of a written test, and pediatric intensive care clinical experience. An advance practice degree and recommendation from a pediatric physician are not required to achieve CCRN certification. Text Reference - p. 1599

The critical care nurse is pursuing certification as a pediatric critical care nurse (CCRN). What will the nurse need to accomplish to achieve this certification? Select all that apply. 1 Registered nurse licensure 2 An advanced practice degree 3 Successful completion of a written test 4 Pediatric intensive care clinical experience 5 Recommendation from a pediatric physician

4 Progressive care units (PCUs), also called intermediate care units, provide a transition between the intensive care unit (ICU) and the general care unit or discharge. Generally, PCU patients are at risk for serious complications, but their risk is lower than that of the ICU patient. Therefore, the nurse anticipates that the patient with a stable blood pressure on a stable dose of a vasoactive drug will be transferred to the PCU. A patient who required a new drip overnight for hypotension and a patient who crashed and required intubation and mechanical ventilation are unstable and the nurse does not anticipate their transfer to the PCU. A preoperative heart transplant patient is often admitted to the PCU. A patient who is 12 hours post transplantation is not expected to be transferred to the PCU at this time. Text Reference - p. 1599

The intensive care unit (ICU) charge nurse is reviewing patient medical records during the overnight shift. Which patient does the nurse anticipate will be transferred to the progressive care unit (PCU)? 1 The patient who required a new drip overnight for hypotension. 2 The patient who was intubated and ventilated after crashing overnight. 3 The patient who is 12 hours postop for a heart and lung transplantation. 4 The patient whose blood pressure is 100/70 mm Hg on a stable dose of a vasoactive drug.

1, 2, 3 ICU patients often require intensive and complicated nursing support related to the use of IV polypharmacy and advanced technology. Prescriptions that are classified as IV polypharmacy include sedation, thrombolytics, and vasopressor titration. Mechanical ventilation and hemodynamic monitoring are advanced technology prescriptions. Text Reference - p. 1600

The intensive care unit (ICU) nurse is providing care to a patient requiring IV polypharmacy. Which prescriptions is this patient receiving? Select all that apply. 1 Sedation 2 Thrombolytics 3 Vasopressor titration 4 Mechanical ventilation 5 Hemodynamic monitoring

4 Negative inspiratory force (NIF) is the amount of negative pressure that a patient is able to generate to initiate spontaneous respirations. An NIF of less than -20 cm H2O is an indicator for weaning but the more negative the number, the better the indication for weaning. Therefore -60 cm H2O is the best indication for weaning. Text Reference - p. 1626

The negative inspiratory force (NIF) is measured in a patient who is on positive pressure ventilation. Among the values given, which NIF value is the best indication for weaning? 1 -30 cm H2O 2 -40 cm H2O 3 -50 cm H2O 4 -60 cm H2O

2 Frank-Starling's law explains the effects of preload and states that the more a myocardial fiber is stretched during filling, the more it shortens during systole and the greater the force of the contraction. Cardiac index (CI) is the measurement of the cardiac output adjusted for body surface area (BSA). It is a more precise measurement of the efficiency of the heart's pumping action. Systemic vascular resistance (SVR) is opposition encountered by the left ventricle to blood flow by the vessels. Pulmonary vascular resistance (PVR) is opposition encountered by the right ventricle to blood flow by the vessels. Text Reference - p. 1604

The nurse educator is preparing a lecture on hemodynamic monitoring. What should the educator use to explain the effects of preload? 1 Cardiac index 2 Frank-Starling's law 3 Systemic vascular resistance 4 Pulmonary vascular resistance

1 Critical care nursing is a specialty dealing with human responses to life-threatening problems. Medial surgical nursing is a specialty dealing with the care of adult patients in a variety of settings. Mental health nursing is a specialty dealing with people of all ages with mental illness or mental distress. Maternal newborn nursing is a specialty dealing with the care of women throughout their pregnancy and childbirth. Text Reference - p. 1598

The nurse educator is teaching a group of nursing students about critical care nursing. Which statement by a student indicates appropriate understanding of this nursing specialty? 1 "Critical care nursing is a specialty dealing with human responses to life-threatening problems." 2 "Critical care nursing is a specialty dealing with the care of adult patients in a variety of settings." 3 "Critical care nursing is a specialty dealing with people of all ages with mental illness or mental distress." 4 "Critical care nursing is a specialty dealing with the care of women throughout their pregnancy and childbirth."

1, 4, 5 Pulmonary artery catheterization helps to monitor and manage the care of patients who are at high risk for hemodynamic compromise. Pulmonary artery catheterization may cause trauma in the blood vessels and worsen symptoms of coagulopathy. Pulmonary artery catheterization increases the risk of trauma in patients with mechanical tricuspid valves and endocardial pacemakers; therefore, it is contraindicated in the patient with coagulopathy, mechanical tricuspid valve, and transvenous pacemaker. Pulmonary artery catheterization is performed in patients with cardiogenic shock and fulminant myocarditis to detect the risk of heart failure. Text Reference - p. 1607

The nurse identifies that pulmonary artery catheterization is contraindicated for patients with a history of what? Select all that apply. 1 Coagulopathy 2 Cardiogenic shock 3 Fulminant myocarditis 4 Endocardial pacemaker 5 Mechanical tricuspid valve

1, 2, 3 Intraaortic balloon therapy destroys platelets and may cause thrombocytopenia. Monitoring hemodynamic parameters helps estimate the loss of platelets and development of thrombus. Enlarged blood clots reduce blood flow to the tissues and result in hypoxemia. Therefore, measurement of oxygen deprivation levels assesses for thrombus formation in the patient. Peripheral neurovascular damage is generally observed as a result of intraaortic balloon therapy and should be monitored. Spinach is rich in vitamin K and increases the formation of clotting factors. Warfarin is an anticoagulant used to prevent future development of blood clots. Spinach and warfarin should be avoided for patients with thrombocytopenia. Text Reference - p. 1611

The nurse is caring for a patient receiving intraaortic balloon therapy. Which nursing interventions are appropriate for this patient? Select all that apply. 1 Monitoring oxygen deprivation levels 2 Monitoring hemodynamic parameters 3 Monitoring neurovascular complications 4 Including spinach in the patient's diet 5 Providing warfarin therapy

4 Intraaortic balloon therapy has potential complications such as site infection, thromboembolism, arterial trauma, hematologic complications, and hemorrhage from the insertion site. Maintaining the head of the bed below 45 degrees helps to prevent breathlessness in the patient in the event of arterial trauma. Monitoring coagulation profiles, hematocrit, and platelet count is beneficial when the patient has a hematologic complication. An occlusive dressing prevents risk of surface infection but is not used to treat arterial trauma in the patient. Heparin helps to prevent thromboembolism but does not treat arterial trauma. Text Reference - p. 1612

The nurse is caring for a patient who has suffered arterial trauma during intraaortic balloon pump (IABP) therapy. What is the appropriate nursing action for this patient? 1 Monitor coagulation profiles 2 Apply an occlusive dressing 3 Administer prophylactic heparin 4 Maintain head of the bed below 45 degrees

1, 3, 4 Administering IV fluids helps thin respiratory secretions and facilitates suctioning. Postural drainage and percussion every two hours helps move secretions into larger airways and promotes the removal through suctioning. Supplemental humidification helps thin secretions and promotes suctioning. The patient has an endotracheal tube and will not be given an oral diet. Instilling normal saline into the endotracheal tube can cause asphyxia. Text Reference - p. 1617

The nurse is caring for a patient with an artificial airway. Which nursing interventions ensure endotracheal tube patency? Select all that apply. 1 Provide adequate hydration 2 Provide gluten rich food 3 Promote postural drainage 4 Provide supplemental humidification 5 Instilling normal saline into the endotracheal tube

1 Preload, afterload, and contractility determine stroke volume (SV). Cardiac output and heart rate are used to determine stroke volume; however, body surface area is used to determine cardiac index. Mean arterial pressure is used to determine afterload, not stroke volume. Cardiac index is a more precise measurement of the efficiency of the heart's pumping action; it is not used to determine stroke volume. Text Reference - p. 1603

The nurse is concerned about a patient's stroke volume. What determines stroke volume? 1 Preload, afterload, and contractility 2 Cardiac output, heart rate, and body surface area 3 Afterload, cardiac output, and mean arterial pressure 4 Cardiac index, mean arterial pressure, and blood pressure

1, 2, 3, 5 Hemodynamic effects of intraaortic balloon pump therapy (IABP) include increased stroke volume leading to warm skin and increased urine output. The decrease in afterload improves breath sounds. Improved stroke volume also improves mentation. The pump has no direct effect on blood pressure regulation. Text Reference - p. 1611

The nurse is evaluating a patient receiving intraaortic balloon pump (IABP) therapy. Which findings indicate that the pump is improving the patient's health status? Select all that apply. 1 Warm and dry skin 2 Urine output 50 mL/hr 3 Breath sounds clear bilaterally 4 Blood pressure 168/88 mm Hg 5 Oriented to person, place, and time

1 Accurate oxygen saturation of hemoglobin (SpO2) measurements may be difficult to obtain on patients who are hypothermic, are receiving vasopressor therapy, or experiencing shock. A body temperature of 95.4° F is hypothermic and is most likely the reason this measurement is difficult to obtain. A cardiac glycoside is not a vasopressor and would not cause this difficulty. A blood pressure of 118/72 mm Hg and heart rate of 72 with occasional ectopy are not manifestations of shock and would not cause this difficulty. Text Reference - p. 1609

The nurse is having difficulty obtaining an accurate oxygen saturation of hemoglobin (SpO2) measurement on a patient. What should the nurse consider as the reason for this difficulty? 1 Body temperature 95.4° F 2 Receiving a cardiac glycoside 3 Blood pressure of 118/72 mm Hg 4 Heart rate of 72 beats/minute with occasional ectopy

2 Normal central venous oxygen saturation or ScvO2/SvO2 is 60% to 80%. It denotes normal oxygen supply and metabolic demand. Any reading out of the normal range can cause danger to the patient. More than 80% denotes increased oxygen supply and decreased oxygen demand. Less than 60% denotes decreased oxygen supply and increased demand. Text Reference - p. 1609

The nurse is monitoring a patient in ICU. Which ScvO2/SvO2reading is a cause of concern in the patient? 1. 68% 2. 54% 3. 72% 4. 78%

10 The formula to calculate the minute ventilation (VE) is: VE = VTf. Therefore, VE = (20) (0.5) = 10 L/min. Text Reference - p. 1626

The nurse is performing a weaning assessment on a patient receiving mechanical ventilation. What is the value of minute ventilation (VE) if the patient's spontaneous respiratory rate (f) is 20 breaths/minute and spontaneous tidal volume (VT) is 0.5 L? Record your answer using a whole number. ___________________L/min

3, 4, 5 Interventions to reduce the risk of infection in a patient with an intraaortic balloon pump (IABP) include covering the site with an occlusive dressing, administering prophylactic antibiotics as prescribed, and using strict aseptic technique with dressing changes. Keeping the lower extremities extended would help prevent arterial trauma caused by insertion or displacement of the balloon. Turning and repositioning every two hours would help prevent the development of stasis pneumonia. Text Reference - p. 1612

The nurse is planning care for a patient receiving intraaortic balloon pump (IABP) therapy. What interventions should be included to reduce this patient's risk of developing an infection? Select all that apply. 1 Keep lower extremities extended 2 Turn and reposition every two hours 3 Cover the site with an occlusive dressing 4 Administer prophylactic antibiotics as prescribed 5 Use strict aseptic technique with dressing changes

3 Stroke volume variation (SVV) is the variation of the arterial pulsation caused by heart-lung interaction. It is a sensitive indicator of preload responsiveness when used on select patients. SVV is used only for patients on controlled mechanical ventilation with a fixed respiratory rate and a fixed tidal volume of 8 mL/kg. SVV is not used on patients who have spontaneous respirations even though an arterial line does need to be in place. The patient may have continuous cardiac monitoring but the patient needs to be intubated rather than provided with oxygen via a face mask. The patient needs to be intubated; however, nasal intubation is not identified as a requirement. It is not identified that the patient needs to be receiving positive end expiration pressure through the ventilator. Text Reference - p. 1607

The nurse is preparing a patient for arterial pressure-based cardiac output (APCO) measuring. What patient criteria must be met before this measuring device can be used to determine the patient's stroke volume variation (SVV)? 1 Spontaneous respirations and placement of an arterial line 2 Continuous cardiac monitoring and application of oxygen via face mask 3 Controlled mechanical ventilation and fixed respiratory rate and tidal volume 4 Nasal intubation and positive end expiration pressure setting on the ventilator

4 When an intensive care unit (ICU) patient exhibits manifestations of delirium it is the nurse's priority to address physiologic factors that could be contributing to the patient's symptoms. Correction of oxygenation by increasing the patient's oxygen is the priority intervention for this patient. Placing a clock in the room and updating the calendar in the room are important when providing care to a patient with delirium but these are not the priorities in the current situation. Administering an opioid pain medication is likely to enhance the clinical manifestations of delirium. Text Reference - p. 1601

The nurse is providing care for a patient who is receiving care in the intensive care unit (ICU). The patient is exhibiting symptoms of delirium. Which will the nurse address on priority when providing care to this patient? 1 Placing a clock in the room 2 Updating the calendar in the room 3 Administering opioid analgesics for pain 4 Administering increased oxygen, per order

1, 3, 4, 5 Allowing a patient's family to be present during resuscitative efforts during a cardiac arrest can help in the grieving process if the patient dies. It also reduces the fear and anxiety that the family is feeling, allows the patient's family to support their loved one, and helps the family overcome any doubts about the patient's condition. Allowing the family to be in the room during resuscitative efforts has not been shown to decrease the likelihood of malpractice lawsuits. Text Reference - p. 1602

The nurse is providing care to a patient in the intensive care unit (ICU) who is being coded due to cardiac arrest. Why would it be appropriate for the patient's family to be present in this situation? Select all that apply. 1 It aids in the grieving process if the patient dies. 2 It decreases the likelihood of malpractice lawsuits. 3 It may reduce the fear and anxiety the family is feeling. 4 It allows the patient's family to support their loved one. 5 It may help the family overcome doubts about the patient's condition.

2 Music therapy is a nonpharmacologic relaxation intervention that can be used to treat the patient anxiety that often occurs in the intensive care unit (ICU) environment. Lorazepam is an appropriate pharmacologic, not nonpharmacologic, intervention for anxiety. A sedation holiday is appropriate to conduct a neurologic exam that is often difficult due to the deep sedation required for intubation and mechanical ventilation. Range-of-motion exercises can help treat the physical manifestations of an injury, but are not relaxation interventions. Text Reference - p. 1601

The nurse is providing care to a patient in the intensive care unit (ICU) who is experiencing anxiety. Which nonpharmacologic relaxation intervention is appropriate for this patient? 1 Lorazepam 2 Music therapy 3 Sedation holiday 4 Range-of-motion exercises

2 Patients with preexisting dementia, such as Alzheimer's disease, are at an increased risk for developing delirium when receiving care in the intensive care unit (ICU). Diabetes mellitus, Parkinson's disease, and multiple sclerosis are not known risk factors for developing delirium. Text Reference - p. 1601

The nurse is providing care to a patient in the intensive care unit (ICU). The patient is currently sedated due to intubation and mechanical ventilation. Which finding in the patient's medical record would place this patient at an increased risk for delirium? 1 Diabetes mellitus 2 Alzheimer's disease 3 Parkinson's disease 4 Multiple sclerosi

3, 5 Medications that are appropriate to induce and maintain sleep for the patient in the intensive care unit (ICU) environment include zolpidem and tamazepam. Propofol is a medication that is used to induce sedation not sleep. Fentanyl and morphine are opioid medications used to treat pain and are not appropriate to induce and maintain sleep for this patient. Text Reference - p. 1601

The nurse is providing care to a patient in the intensive care unit (ICU). Which pharmacologic interventions are appropriate to induce and maintain sleep? Select all that apply. 1 Propofol 2 Fentanyl 3 Zolpidem 4 Morphine 5 Temazepam

2 Tachypnea and disorientation are early and subtle signs of deterioration. The rapid response team (RRT) brings rapid and immediate care to unstable patients in non-critical care units. While assessing pain, documenting the data in the medical record, and administering prescribed medications such as antihypertensive medications are all appropriate actions, they are not the priority nursing actions in this situation. Text Reference - p. 1599

The nurse is providing care to a postoperative patient on a medical-surgical unit. The patient is experiencing tachypnea and becomes disoriented at times. Which is the priority action by the nurse in this situation? 1 Assessing the current level of pain 2 Activating the rapid response team 3 Documenting the data in the medical record 4 Administering the prescribed antihypertensive medication

1 Nonverbal communication is important when providing care to an unresponsive patient in the intensive care unit (ICU). High levels of procedure-related touch and lower levels of comfort-related touch often characterize the ICU environment. The nurse would encourage the patient's spouse to touch and talk with her husband even if he is unresponsive. There is no way of knowing how aware the patient is of the surrounding when sedated. While bringing items from home for comfort are important, this is not the most appropriate statement by the nurse. Telling the spouse to sit where she will not be in the way of providing care is not therapeutic. Text Reference - p. 1601

The nurse is providing care to an unresponsive patient in the intensive care unit (ICU). The patient's spouse is at the bedside and states, "I just want him to know that I am here with him." Which statement by the nurse is most appropriate? 1 "You should talk and touch you husband whenever you visit." 2 "Your husband is so sedated he is not aware of his surroundings." 3 "You can bring items from home to make your husband more comfortable." 4 "Please sit where you will not be in the way as we provide care to your husband."

4 The patient with a collapsed lung and multiple injuries sustained in a car accident would be admitted to the trauma intensive care unit (ICU). Patients with minor acute injuries or those who are not expected to recover from an illness is usually not admitted to an ICU. Therefore, the ICU is not used to manage a fractured wrist; to prolong the natural process of death such as the patient with stage IV metastatic breast cancer; or for the patient in a persistent coma or vegetative state. Text Reference - p. 1600

The nurse is providing care to several patients in the emergency department (ED). Which patient does the nurse anticipate will be admitted to the intensive care unit (ICU)? 1 A patient with a fractured wrist 2 A patient with stage IV metastatic breast cancer 3 A patient in a permanent vegetative state who has a urinary tract infection 4 The patient with a collapsed lung and multiple injuries sustained in a car accident

4 The nurse should advise the caregiver to avoid external rotation of the patient's hip; this movement can be avoided by properly positioning the patient and by the use of specialized mattresses and beds. Simple maneuvers such as arm circles, knee bends and quadriceps setting should be performed, because they maintain the muscle tone in the upper and lower extremities of the patient. Text Reference - p. 1624

The nurse is teaching the patient's caregiver about receiving positive pressure ventilation. What movements should the nurse tell the caregiver to avoid doing to the patient? 1 Arm circles 2 Knee bends 3 Quadriceps setting 4 External rotation of the hip

2, 3, 4 When enteral nutrition is contraindicated in patients, the primary health care provider would prescribe parenteral nutrition to provide adequate nutrients to the patients. Patients with gastrointestinal disorders such as pancreatitis, paralytic ileus, and severe diarrhea will receive parenteral nutrition. Enteral nutrition is not contraindicated in the patient with arthritis and hypothyroidism. Text Reference - p. 1600

The nurse reviews medical records of several patient and concludes that which patients are appropriate candidates for parenteral nutrition? Select all that apply. 1 A patient with arthritis 2 A patient with pancreatitis 3 A patient with paralytic ileus 4 A patient with severe diarrhea 5 A patient with hypothyroidism

2, 3 Providing culturally competent care to critically ill patients and caregivers is challenging. The nurse who wants to provide culturally competent care to critically ill patients in the intensive care unit (ICU) should ask the patient or family members who is wanted in the room at the time of death. The nurse should also ask the patient or family members about cultural traditions regarding death and dying. The nurse should not prioritize cultural needs over physiologic needs. Often physiologic needs are the priority in the ICU. The nurse should not assume that the patient follows cultural customs for the documented ethnicity. The nurse should not tell the family that last rites are not possible and should advocate for the patient to receive last rites if this is the patient's wish. Text Reference - p. 1602

The nurse wants to provide culturally competent care to patients requiring care in the intensive care unit (ICU). Which actions by the nurse are appropriate? Select all that apply. 1 Prioritizing cultural needs over physiologic needs 2 Asking the patient who he or she wants in the room at the time of death 3 Asking the family about cultural traditions regarding death and dying 4 Assuming that the patient follows cultural customs for the documented ethnicity 5 Telling the family members it is not possible for last rites to be administered

2 Tidal volume is the volume of gas delivered to a patient during each ventilator breath. The number of breaths the ventilator delivers per minute is called the respiratory rate. The positive pressure used to augment the patient's inspiratory pressure is called pressure support. The positive pressure applied at the end of expiration of ventilator breaths is called positive end-expiratory pressure. Text Reference - p. 1619

The nurse working in a critical care unit understands that tidal volume is an important setting in a mechanical ventilator. Which statement appropriately describes tidal volume? 1 Number of breaths the ventilator delivers per minute 2 Volume of gas delivered to patient during each ventilator breath 3 Positive pressure used to augment patient's inspiratory pressure 4 Positive pressure applied at the end of expiration of ventilator breaths

3 The most commonly used mechanical circulatory assist device is the IABP, and it is used to decrease ventricular workload, increase myocardial perfusion, and augment circulation. Cardiopulmonary bypass provides circulation during open heart surgery. It is not used as an assist device after surgery. ICG is a noninvasive method to obtain cardiac output and assess thoracic fluid status. CVP measurement is an invasive measurement of right ventricular preload and reflects fluid volume problems. Text Reference - p. 1611

The patient has developed cardiogenic shock after a left anterior descending myocardial infection. Which circulatory assist device should the nurse expect to use for this patient? 1 Cardiopulmonary bypass 2 Impedance cardiography (ICG) 3 Intraaortic balloon pump (IABP) 4 Central venous pressure (CVP) measuremen

A nurse is weaning a 68-kg patient who has chronic obstructive pulmonary disease (COPD) from mechanical ventilation. Which patient assessment finding indicates that the weaning protocol should be stopped? a. The patient's heart rate is 97 beats/min. b. The patient's oxygen saturation is 93%. c. The patient respiratory rate is 32 breaths/min. d. The patient's spontaneous tidal volume is 450 mL.

The patient respiratory rate is 32 breaths/min. Tachypnea is a sign that the patient's work of breathing is too high to allow weaning to proceed. The patient's heart rate is within normal limits, but the nurse should continue to monitor it. An O2 saturation of 93% is acceptable for a patient with COPD. A spontaneous tidal volume of 450 mL is within the acceptable range.

3 Critical care nurses provide care for patients with acute problems who are unstable. Patients with acute problems who are stable are often cared for on a medical-surgical unit versus the critical care unit. Patients with chronic problems who are stable may be cared for in the community setting. Patients with chronic problems who are unstable may require care in a rehabilitation or medical-surgical setting. Text Reference - p. 1598

The seasoned nurse is orienting a novice nurse to the critical care unit. When teaching the novice nurse about critical care nursing, which statement is most appropriate by the seasoned nurse? 1 "We care for patients with acute problems who are stable." 2 "We care for patients with chronic problems who are stable." 3 "We care for patients with acute problems who are unstable." 4 "We care for patients with chronic problems who are unstable."

4 Pulmonary vascular resistance (PVR) is calculated using the pulmonary artery mean pressure (PAMR) minus the pulmonary artery wedge pressure (PAWP), multiplying by 80 and dividing by the cardiac output (CO). Systolic and diastolic blood pressures are used to determine mean arterial pressure (MAP). Stroke volume (SV) and right ventricular ejection fraction (RVEF) are used to determine right ventricular end-diastolic volume (RVEDV). Mean arterial pressure (MAP), central venous pressure (CVP), and cardiac output (CO) are used to determine systemic vascular resistance (SVR). Text Reference - p. 1603

To determine a patient's peripheral vascular resistance (PVR) what hemodynamic parameters should the nurse use? 1 Systolic and diastolic blood pressures 2 Stroke volume and right ventricular ejection fraction 3 Mean arterial pressure, central venous pressure, and cardiac output 4 Pulmonary artery mean pressure, pulmonary artery wedge pressure, and cardiac output

3 To obtain an accurate central venous pressure (CVP) reading, the nurse should ensure that the patient is supine and in horizontal position so the zero reference point at the level of the right atrium (also known as the phlebostatic axis) is level with the zero mark on the manometer. If a patient is unable to lie flat, the manometer should be positioned so the zero reference is at the level of the right atrium and the degree of head elevation noted so there is consistency across measurements. Right or left side-lying and supine positions with the head of the bed elevated 45 degrees are all incorrect for obtaining a CVP reading. Text Reference - p. 1605

To obtain an accurate central venous pressure reading with a central venous catheter, a nurse should place the patient in what position? 1 Left side-lying 2 Right side-lying 3 Supine and horizontal 4 Supine with head of bed elevated 45 degrees

1 Both negative pressure ventilation and positive pressure ventilation involve passive expiration. Negative pressure ventilation does not require an artificial airway. Positive pressure ventilation may require invasive mechanical ventilation through an artificial airway. Positive pressure ventilation is primarily used with acutely ill patients; negative pressure ventilation is not routinely used for acutely ill patients. In positive pressure ventilation, intrathoracic pressure is raised during lung inflation but in negative pressure ventilation, intrathoracic pressure is decreased during lung inflation. Text Reference - p. 1618

What is the similarity between negative pressure ventilation and positive pressure ventilation? 1 Both involve passive expiration. 2 Both require an artificial airway. 3 Both are routinely used for acutely ill patients. 4 Both raise intrathoracic pressure during lung inflation.

3 Negative inspiratory force is used to assess the muscle strength in a patient with positive pressure ventilation. Vital capacity, minute ventilation and rapid shallow breathing index are used to assess the muscle endurance. Test-Taking Tip: Multiple-choice questions can be challenging because students think that they will recognize the right answer when they see it or that the right answer will somehow stand out from the other choices. This is a dangerous misconception. The more carefully the question is constructed, the more each of the choices will seem like the correct response. Text Reference - p. 1626

What measurement is used to assess muscle strength in a patient with positive pressure ventilation? 1 Vital capacity 2 Minute ventilation 3 Negative inspiratory force 4 Rapid shallow breathing index

75 Rapid shallow breathing index is equal to respiratory rate/tidal volume; therefore, 30/0.4 = 75. Text Reference - p. 1626

What will be the rapid shallow breathing index for a patient with a tidal volume of 0.4 L and respiratory rate of 30/minute? Record your answer using a whole number. ___ L

3 Systemic vascular resistance is an index of left ventricular afterload. Central venous pressure is an index of preload. Pulmonary arterial pressure and peripheral vascular resistance are indices of right ventricular afterload. Text Reference - p. 1604

Which hemodynamic value should the nurse use to determine a patient's left ventricular afterload? 1 Central venous pressure 2 Pulmonary arterial pressure 3 Systemic vascular resistance 4 Peripheral vascular resistance

3 A dynamic response test is performed every 8 to 12 hours, as well as when the system is opened to air or the accuracy of the measurements is questioned. It involves activating the fast flush and checking that the equipment reproduces a distortion-free signal. A square wave indicates a normal response and requires no further action. This waveform does not indicate that the line needs to be flushed. Nor does it indicate that zero needs to be reconfirmed or that the wrist needs to be repositioned. Text Reference - p. 1605

When performing a dynamic response test, the nurse observes the following tracing. What action should the nurse perform based on this tracing?

1 Prior to the family entering the intensive care unit (ICU) to visit a family member who is critically ill the nurse should provide the family with a description of what to expect regarding the patient's appearance and the equipment that is being used to provide care to their family member. The nurse should accompany the family into the patient's room; it is not appropriate for the nurse to ask the physician to do this, nor is it appropriate for the family to enter the room alone. Although it is important to instruct the family on what to expect, the family should be encouraged to touch and speak to the patient. Text Reference - p. 1602

Which action by the nurse is most appropriate when bringing a family member of a critically ill patient into the intensive care unit (ICU) for the first time? 1 Give a description of what to expect 2 Ask the physician to accompany the family 3 Allow the family member to enter the room alone 4 Instruct the family not to touch and speak to the patient

1, 2, 3, 4 Caregivers play a valuable role in the intensive care unit (ICU) patient's recovery because they provide the patient with support. They are also a link to the patient's personal life, help the patient with activities of daily living, and function as the decision maker during the hospitalization. Although the caregiver may make financial decision for the patient during the hospitalization, this is not an important role the caregiver plays in the patient's recovery. Text Reference - p. 1601

Which are the reasons why caregivers play a valuable role in the intensive care unit (ICU) patient's recovery? Select all that apply. 1 They provide loving support for the patient. 2 They provide a link to the patient's personal life. 3 They help the patient with activities of daily living. 4 They function as the patient's decision-maker during the hospitalization. 5 They make the financial decision for the patient during the hospitalization.

4 The electronic or teleICU assists the bedside ICU team by monitoring the patient from a remote location using informatics. The ICU, CCU, and PICU are traditional critical care units. Text Reference - p. 1599

Which type of critical care unit uses informatics to monitor a critically ill patient from a remote location? 1 Intensive care unit (ICU) 2 Coronary care unit (CCU) 3 Pediatric intensive care unit (PICU) 4 Electronic intensive care unit (teleICU)

A patient who is receiving positive pressure ventilation is scheduled for a spontaneous breathing trial (SBT). Which finding by the nurse is most likely to result in postponing the SBT? a. New ST segment elevation is noted on the cardiac monitor. b. Enteral feedings are being given through an orogastric tube. c. Scattered rhonchi are heard when auscultating breath sounds. d. hydromorphone (Dilaudid) is being used to treat postoperative pain.

a. New ST segment elevation is noted on the cardiac monitor. Myocardial ischemia is a contraindication for ventilator weaning. The ST segment elevation is an indication that weaning should be postponed until further investigation and/or treatment for myocardial ischemia can be done. Ventilator weaning can proceed when opioids are used for pain management, abnormal lung sounds are present, or enteral feedings are being used.

40. A patient is to be discharged home with mechanical ventilation. Before discharge, it is most important for the nurse to a. teach the caregiver to care for the patient with a home ventilator. b. help the caregiver plan for placement of the patient in a long-term care facility. c. stress the advantages for the patient in being cared for in the home environment. d. have the caregiver arrange for around-the-clock home health nurses for the first several weeks.

a. Rationale: Care of a ventilator-dependent patient in the home requires that the caregiver know how to manage the ventilator and take care of the patient on it. The nurse should ensure that caregivers understand the potential sacrifices they may have to make and the impact that home mechanical ventilation will have over time, before final decisions and arrangements are made. Placement in long- term care facilities is not usually necessary unless the caregiver can no longer manage the care or the patient's condition deteriorates.

A pressure (PEEP) of 10 cm H2O. A sign that alerts the nurse to undesirable effects of increased airway and thoracic pressure is a. decreased BP. b. decreased PaO2. c. increased crackles. d. decreased spontaneous respirations.

a. Rationale: Positive-pressure ventilation, especially with end-expiratory pressure, increases intrathoracic pressure with compression of thoracic vessels, resulting in decreased venous return to the heart, decreased left ventricular end-diastolic volume (preload), decreased CO, and lowered BP. None of the other factors is related to increased intrathoracic pressure.

In preparing a patient in the ICU for oral endotracheal intubation, the nurse a. places the patient supine with the head extended and neck flexed. b. tells the patient that the tongue must be extruded while the tube is inserted. c. positions the patient supine with the head hanging over the edge of the bed to align the mouth and trachea. d. informs the patient that while it will not be possible to talk during insertion of the tube, speech will be possible after it is correctly placed.

a. Rationale: The patient is positioned with the mouth, pharynx, and trachea in direct alignment, with the head extended in the "sniffing position," but the head must not hang over the edge of the bed. The patient may be asked to extrude the tongue during nasal intubation. Speaking is not possible during intubation or while the tube is in place because the tube splits the vocal cords.

Identify two precautions the nurse should take during mouth care and repositioning of an oral ET tube to prevent and detect tube dislodgement.

a. Use two nurses: one to hold the tube while it is untaped or the holder is loosened, and another to perform care. b. After completion of care, confirm the presence of bilateral breath sounds to ensure that the position of the tube was not changed and reconfirm cuff pressure.

A patient who is orally intubated and receiving mechanical ventilation is anxious and is "fighting" the ventilator. Which action should the nurse take next? a. Verbally coach the patient to breathe with the ventilator. b. Sedate the patient with the ordered PRN lorazepam (Ativan). c. Manually ventilate the patient with a bag-valve-mask device. d. Increase the rate for the ordered propofol (Diprivan) infusion.

a. Verbally coach the patient to breathe with the ventilator. The initial response by the nurse should be to try to decrease the patient's anxiety by coaching the patient about how to coordinate respirations with the ventilator. The other actions may also be helpful if the verbal coaching is ineffective in reducing the patient's anxiety.

Identify five problems associated with inadequate nutrition in the patient receiving prolonged mechanical ventilation.

a. anemia resulting in poor O2 transport; b. decreased respiratory strength; c. delayed weaning; d. decreased resistance to infection; e. prolonged recovery

Identify which of the common three reasons patients are admitted to the ICU apply in the following situations. a. Patient with diabetic ketoacidosis b. Patient with nondisplaced skull fracture who is alert and oriented c. Postoperative patient with mitral valve replacement d. Comatose patient who had an anaphylactic reaction with cardiopulmonary arrest at home with reestablishment of cardiac function

a. physiologically unstable; b. risk for serious complications; c. risk for serious complications; d. intensive nursing support

A patient with an oral ET tube has a nursing diagnosis of risk for aspiration related to presence of artificial airway. Appropriate nursing interventions for this patient are to (select all that apply) a. assess gag reflex. b. ensure the cuff is properly inflated. c. suction the patient's mouth frequently. d. raise the head of the bed 30 to 45 degrees unless the patient is unstable. e. keep the ventilator tubing cleared of condensed water.

b, c, d. Rationale: Because the patient with an ET tube cannot protect the airway from aspiration and cannot swallow, the cuff should always be inflated and the head of the bed (HOB) elevated while the patient is receiving tube feedings or mouth care is being performed. The HOB elevated 30 to 45 degrees helps reduce risk. The mouth and oropharynx should be suctioned with Yankauer or tonsil suction to remove accumulated secretions that cannot be swallowed. Clearing the ventilatory tubing of condensed water is important to prevent respiratory infection.

A patient is receiving 35% O2 via a Venturi mask. To ensure the correct amount of O2 delivery, which action by the nurse is important? a. Teach the patient to keep the mask on during meals. b. Keep the air entrainment ports clean and unobstructed. c. Give a high enough flow rate to keep the bag from collapsing. d. Drain moisture condensation from the corrugated tubing every hour.

b. Keep the air entrainment ports clean and unobstructed. The air entrainment ports regulate the O2 percentage delivered to the patient, so they must be unobstructed. The other options refer to other types of O2 devices. A high O2 flow rate is needed when giving O2by partial rebreather or nonrebreather masks. Draining O2 tubing is necessary when caring for a patient receiving mechanical ventilation. The mask can be removed or changed to a nasal cannula at a prescribed setting when the patient eats.

The nurse determines that alveolar hypoventilation is occurring in a patient on a ventilator when a. the patient develops cardiac dysrhythmias. b. auscultation reveals an air leak around the ET cuff. c. ABG results show a PaCO2 of 32 mm Hg and a pH of 7.47. d. the patient tries to breathe faster than the ventilator setting.

b. Rationale: A leaking cuff can lower tidal volume or respiratory rates. An SIMV rate that is too low, the presence of lung secretions, or obstruction can decrease tidal volume. A decreased PaCO2 and increased pH indicate a respiratory alkalosis from hyperventilation, and cardiac dysrhythmias can occur with either hyperventilation or hypoventilation.

When weaning a patient from a ventilator, the nurse plans a. to decrease the delivered FIO2 concentration. b. intermittent trials of spontaneous ventilation followed by ventilatory support to provide rest. c. substitution of ventilator support with a manual resuscitation bag if the patient becomes hypoxemic. d. to implement weaning procedures around the clock until the patient does not experience ventilatory fatigue.

b. Rationale: A variety of ventilator weaning methods is used, but all should provide weaning trials with adequate rest between weaning trials to prevent respiratory muscle fatigue. Weaning is usually carried out during the day, with the patient ventilated at night until there is sufficient spontaneous ventilation without excess fatigue. In all methods, patients usually require a 10% increase in fraction of inspired oxygen (FiO2) to maintain arterial oxygen tension. If the patient becomes hypoxemic, ventilator support is indicated.

Before taking hemodynamic measurements, the monitoring equipment must be referenced by a. confirming that when pressure in the system is zero, the equipment reads zero. b. positioning the stopcock nearest the transducer level with the phlebostatic axis. c. placing the transducer on the left side of the chest at the fourth intercostal space. d. placing the patient in a left lateral position with the transducer level with the top surface of the mattress.

b. Rationale: Referencing hemodynamic monitoring equipment means positioning the monitoring equipment so that the zero reference point is at the vertical level of the left atrium of the heart. The port of the stopcock nearest the transducer is placed at the phlebostatic axis, the external landmark of the left atrium. The phlebostatic axis is the intersection of two planes: a horizontal line midchest, halfway between the outermost anterior and posterior surfaces, transecting a vertical line through the fourth intercostal space at the sternum.

A patient has SvO2 of 52%, CO of 4.8 L/min, SpO2 of 95%, and unchanged hemoglobin level. The nurse should assess the patient for a. dysrhythmias. b. pain on movement. c. pulmonary edema. d. signs of septic shock.

b. Rationale: The normal mixed venous oxygen saturation of 60% to 80% becomes decreased with decreased arterial oxygenation, low CO, low hemoglobin, or increased oxygen consumption. With normal CO, arterial oxygenation, and hemoglobin, the factor that is responsible for decreased SvO2 is increased oxygen consumption, which can result from increased metabolic rate, pain, movement, or fever.

A nursing intervention indicated for the patient in the ICU who has a nursing diagnosis of anxiety related to ICU environment and sensory overload is a. providing flexible visiting schedules for caregivers. b. eliminating unnecessary alarms and overhead paging. c. administering sedatives or psychotropic drugs to promote rest. d. allowing the patient to do as many self-care activities as possible.

b. Rationale: When anxiety in the ICU patient is related to the environment, which has unfamiliar equipment, high noise and light levels, and an intense pace of activity, which leads to sensory overload, the nurse should eliminate as much of this source of stress as possible by muting phones, limiting overhead paging, setting alarms appropriate to the patient's condition, and eliminating unnecessary alarms. Offering flexible visiting schedules for family members and providing as much autonomy in decisions about care as possible are indicated when impaired communication and loss of control contribute to the anxiety. Use of sedation to reduce anxiety should be carefully evaluated and implemented when nursing measures are not effective.

The nurse educator is evaluating the performance of a new registered nurse (RN) who is providing care to a patient who is receiving mechanical ventilation with 15 cm H2O of peak end-expiratory pressure (PEEP). Which action indicates that the new RN is safe? a. The RN plans to suction the patient every 1 to 2 hours. b. The RN uses a closed-suction technique to suction the patient. c. The RN tapes the connection between the ventilator tubing and the ET. d. The RN changes the ventilator circuit tubing routinely every 48 hours.

b. The RN uses a closed-suction technique to suction the patient. The closed-suction technique is used when patients require high levels of PEEP (>10 cm H2O) to prevent the loss of PEEP that occurs when disconnecting the patient from the ventilator. Suctioning should not be scheduled routinely, but it should be done only when patient assessment data indicate the need for suctioning. Taping connections between the ET and ventilator tubing would restrict the ability of the tubing to swivel in response to patient repositioning. Ventilator tubing changes increase the risk for ventilator-associated pneumonia and are not indicated routinely.

To verify the correct placement of an oral endotracheal tube (ET) after insertion, the best initial action by the nurse is to a. obtain a portable chest x-ray. b. use an end-tidal CO2 monitor. c. auscultate for bilateral breath sounds. d. observe for symmetrical chest movement.

b. use an end-tidal CO2 monitor. End-tidal CO2 monitors are currently recommended for rapid verification of ET placement. Auscultation for bilateral breath sounds and checking chest expansion are also used, but they are not as accurate as end-tidal CO2 monitoring. A chest x-ray confirms the placement but is done after the tube is secured.

The nurse notes thick, white secretions in the endotracheal tube (ET) of a patient who is receiving mechanical ventilation. Which intervention will most directly treat this finding? a. Reposition the patient every 1 to 2 hours. b. Increase suctioning frequency to every hour. c. Add additional water to the patient's enteral feedings. d. Instill 5 mL of sterile saline into the ET before suctioning.

c. Add additional water to the patient's enteral feedings. ANS: C Because the patient's secretions are thick, better hydration is indicated. Suctioning every hour without any specific evidence for the need will increase the incidence of mucosal trauma and would not address the etiology of the ineffective airway clearance. Instillation of saline does not liquefy secretions and may decrease the SpO2. Repositioning the patient is appropriate but will not decrease the thickness of secretions.

The nurse notes that a patient's endotracheal tube (ET), which was at the 22-cm mark, is now at the 25-cm mark, and the patient is anxious and restless. Which action should the nurse take next? a. Check the O2 saturation. b. Offer reassurance to the patient. c. Listen to the patient's breath sounds. d. Notify the patient's health care provider.

c. Listen to the patient's breath sounds. The nurse should first determine whether the ET tube has been displaced into the right mainstem bronchus by listening for unilateral breath sounds. If so, assistance will be needed to reposition the tube immediately. The other actions are also appropriate, but detection and correction of tube malposition are the most critical actions.

After change-of-shift report on a ventilator weaning unit, which patient should the nurse assess first? a. Patient who failed a spontaneous breathing trial and has been placed in a rest mode on the ventilator b. Patient who is intubated and has continuous partial pressure end-tidal CO2 (PETCO2) monitoring c. Patient who was successfully weaned and extubated 4 hours ago and has no urine output for the last 6 hours d. Patient with a central venous O2 saturation (ScvO2) of 69% while on bilevel positive airway pressure (BiPAP)

c. Patient who was successfully weaned and extubated 4 hours ago and has no urine output for the last 6 hours The decreased urine output may indicate acute kidney injury or that the patient's cardiac output and perfusion of vital organs have decreased. Any of these causes would require rapid action. The data about the other patients indicate that their conditions are stable and do not require immediate assessment or changes in their care. Continuous PETCO2 monitoring is frequently used when patients are intubated. The rest mode should be used to allow patient recovery after a failed SBT, and an ScvO2 of 69% is within normal limits.

A comatose patient with a possible cervical spine injury is intubated with a nasal endotracheal tube. The nurse recognizes that in comparison with an oral endotracheal tube, a nasal tube a. requires the placement of a bite block. b. is more likely to cause laryngeal trauma. c. requires greater respiratory effort in breathing. d. provides for easier suctioning and secretion removal.

c. Rationale: A nasal endotracheal (ET) tube is longer and smaller in diameter than an oral ET tube, creating more airway resistance and increasing the work of breathing. Suctioning and secretion removal are also more difficult with nasal ET tubes, and they are more subject to kinking than are oral tubes. Oral tubes require a bite block to stop the patient from biting the tube and may cause more laryngeal damage because of their larger size.

To prevent arterial trauma during the use of the IABP, the nurse should a. reposition the patient every 2 hours. b. check the site for bleeding every hour. c. prevent hip flexion of the cannulated leg. d. cover the insertion site with an occlusive dressing.

c. Rationale: Because the IABP is inserted into the femoral artery and advanced to the descending thoracic aorta, compromised distal extremity circulation is common and requires that the cannulated extremity be extended at all times. Repositioning the patient is limited to side-lying or supine positions with the head of the bed elevated no more than 30 to 45 degrees. Assessment for bleeding is important because the IABP may cause platelet destruction, and occlusive dressings are used to prevent site infection.

In preparing the patient for insertion of a pulmonary artery catheter, the nurse a. obtains informed consent from the patient. b. places the patient in high Fowler's position. c. ensures that the patient has continuous ECG monitoring. d. performs an Allen test to confirm adequate ulnar artery perfusion.

c. Rationale: During insertion of a pulmonary artery catheter, it is necessary to monitor the ECG continuously because of the risk for dysrhythmias, particularly when the catheter reaches the right ventricle. It is the health care provider's responsibility to obtain informed consent regarding the catheter insertion. During the catheter insertion, the patient is placed supine with the head of the bed flat. An Allen test to confirm adequate ulnar artery perfusion is performed before insertion of an arterial catheter in the radial artery for arterial pressure monitoring.

A patient receiving mechanical ventilation is very anxious and agitated, and neuromuscular blocking agents are used to promote ventilation. The nurse recognizes that a. the patient will be too sedated to be aware of the details of care. b. caregivers should be encouraged to provide stimulation and diversion. c. the patient should always be addressed and explanations of care given. d. communication will not be possible with the use of neuromuscular blocking agents.

c. Rationale: Neuromuscular blocking agents produce a paralysis that facilitates ventilation, but they do not sedate the patient. It is important for the nurse to remember that the patient can hear, see, think, and feel and should be addressed and given explanations accordingly. Communication with the patient is possible, especially from the nurse, but visitors for an anxious and agitated patient should provide a calming, restful effect on the patient.

Although his oxygen saturation is above 92%, an orally intubated, mechanically ventilated patient is restless and very anxious. What intervention should be used first to decrease the risk of accidental extubation? a. Obtain an order and apply soft wrist restraints. b. Remind the patient that he needs the tube inserted to breathe. c. Administer sedatives and have a caregiver stay with the patient. d. Move the patient to an area close to the nurses' station for closer observation.

c. Rationale: Sedation may be appropriate as well as having someone the patient knows at the bedside talking to him; reassuring him may decrease his anxiety and calm him. The other methods may need to be used. Restraints will need ongoing and frequent assessment of need. Reminding the patient may help, but it may not be enough to prevent the patient from pulling the tube if the patient becomes extremely anxious.

The critical care nurse includes caregivers of the patient in the ICU as part of the health care team primarily because a. the costs of critical care will affect the entire family. b. caregivers are responsible for making health care decisions for the patient. c. the extent of the caregivers' involvement affects the patient's clinical course. d. caregivers who are ignored are more likely to question the patient's quality of care.

c. Rationale: The caregivers of the critically ill patient are very important in the recovery and well-being of the patient, and the extent to which the family is involved and supported affects the patient's clinical course. Although the cost of planning and providing critical care is a concern to caregivers, it is not the major reason caregivers are included in the patient's care. Caregivers may be responsible for making decisions about the patient's care only when the patient is unable to make personal decisions. Most caregivers have questions regarding the patient's quality of care because of anxiety and lack of information about the patient's condition.

A patient has an oral endotracheal (ET) tube inserted to relieve an upper airway obstruction and to facilitate secretion removal. The first responsibility of the nurse immediately following placement of the tube is to a. suction the tube to remove secretions. b. secure the tube to the face with adhesive tape. c. place an end tidal CO2 detector on the ET tube. d. assess for bilateral breath sounds and symmetric chest movement.

c. Rationale: The first action of the nurse is to use an end-tidal CO2 detector. If no CO2 is detected, the tube is in the esophagus. The second action by the nurse following ET intubation is to auscultate the chest to confirm bilateral breath sounds and observe to confirm bilateral chest expansion. If this evidence is present, the tube is secured and connected to an O2 source. Then the placement is confirmed immediately with x-ray, and the tube is marked where it exits the mouth. Then the patient should be suctioned as needed.

The nurse uses the minimal occluding volume to inflate the cuff on an endotracheal tube to minimize the incidence of a. infection. b. hypoxemia. c. tracheal necrosis. d. accidental extubation.

c. Rationale: The minimal occluding volume (MOV) involves adding air to the ET tube cuff until no leak is heard at peak inspiratory pressure but ensures that minimal pressure is applied to the tracheal wall to prevent pressure necrosis of the trachea. The minimal occluding volume should apply between 20 to 25 mm Hg of pressure on the trachea to prevent injury. The cuff does not secure the tube in place but rather prevents escape of ventilating gases through the upper airway.

Which assessment finding obtained by the nurse when caring for a patient receiving mechanical ventilation indicates the need for suctioning? a. The patient was last suctioned 6 hours ago. b. The patient's oxygen saturation drops to 93%. c. The patient's respiratory rate is 32 breaths/min. d. The patient has occasional audible expiratory wheezes.

c. The patient's respiratory rate is 32 breaths/min. The increase in respiratory rate indicates that the patient may have decreased airway clearance and requires suctioning. Suctioning is done when patient assessment data indicate that it is needed and not on a scheduled basis. Occasional expiratory wheezes do not indicate poor airway clearance, and suctioning the patient may induce bronchospasm and increase wheezing. An O2 saturation of 93% is acceptable and does not suggest that immediate suctioning is needed.

The nurse recognizes that a factor commonly responsible for sodium and fluid retention in the patient on mechanical ventilation is a. increased ADH release. b. increased release of atrial natriuretic factor. c. increased insensible water loss via the airway. d. decreased renal perfusion with release of renin.

d. Rationale: Decreased CO associated with positive-pressure ventilation and positive end- expiratory pressure (PEEP) results in decreased renal perfusion, release of renin, and increased aldosterone secretion, which causes sodium and water retention. ADH may be released because of stress, but ADH is responsible only for water retention, and increased intrathoracic pressure decreases, not increases, the release of atrial natriuretic factor, causing sodium retention. There is decreased, not increased, insensible water loss via the airway during mechanical ventilation.

While suctioning the ET tube of a spontaneously breathing patient, the nurse notes that the patient develops bradycardia with premature ventricular contractions. The nurse should a. stop the suctioning and assess the patient for spontaneous respirations. b. attempt to resuction the patient with reduced suction pressure and pass time. c. stop the suctioning and ventilate the patient with slow, small-volume breaths using a bag-valve-mask (BVM) device. d. stop suctioning and ventilate the patient with a BVM device with 100% oxygen until the HR returns to baseline

d. Rationale: If serious dysrhythmias occur during suctioning, the suctioning should be stopped, and the patient should be slowly ventilated via BVM with 100% oxygen until the dysrhythmia subsides. Patients with bradycardia should not be suctioned excessively. Ventilation of the patient with slow, small-volume breaths using the BVM is performed when severe coughing results from suctioning.

A primary difference in the skills of a certified critical care nurse compared with nurses certified in medical-surgical nursing is an ability to a. diagnose and treat life-threatening diseases. b. detect and manage early complications of health problems. c. provide intensive psychologic support to the patient and family. d. use advanced technology to assess and maintain physiologic function.

d. Rationale: One of the primary characteristics of critical care nurses that is different from those of generalist medical-surgical nurses is the use of advanced technology to measure physiologic parameters accurately to manage life-threatening complications. All nursing addresses human responses to health problems and requires knowledge of physiology, pathophysiology, pharmacology, and psychologic support to the patient and family. Diagnosis and treatment of life-threatening diseases are roles of medicine.

The use of the intraaortic balloon pump (IABP) would be indicated for the patient with a. an insufficient aortic valve. b. a dissecting thoracic aortic aneurysm. c. generalized peripheral vascular disease. d. acute myocardial infarction with heart failure.

d. Rationale: The counterpulsation of the intraaortic balloon pump (IABP) increases diastolic arterial pressure, forcing blood back into the coronary arteries and main branches of the aortic arch, increasing coronary artery perfusion pressure and blood flow to the myocardium. The balloon pump also causes a drop in aortic pressure just before systole, decreasing afterload and myocardial oxygen consumption. These effects make the IABP valuable in treating unstable angina, acute myocardial infarction with heart failure, cardiogenic shock, and a variety of surgical heart situations. Its use is contraindicated in incompetent aortic valves, dissecting aortic aneurysms, and generalized peripheral vascular disease.

A patient who is hemodynamically stable has an order to wean the IABP. The nurse should a. decrease the augmentation pressure to zero. b. stop the machine since hemodynamic parameters are satisfactory. c. stop the infusion flow through the catheter when weaning is initiated. d. change the pumping ratio from 1:1 to 1:2 or 1:3 until the balloon is removed.

d. Rationale: Weaning from the IABP involves reducing the pumping to every second or third heartbeat until the IABP catheter is removed. The pumping and infusion flow are continued to reduce the risk for thrombus formation around the catheter until it is removed.

The nurse observes a PAWP waveform on the monitor when the balloon of the patient's pulmonary artery catheter is deflated. The nurse recognizes that a. the patient is at risk for embolism because of occlusion of the catheter with a thrombus. b. the patient is developing pulmonary edema that has increased the pulmonary artery pressure. c. the patient is at risk for an air embolus because the injected air cannot be withdrawn into the syringe. d. the catheter must be immediately repositioned to prevent pulmonary infarction or pulmonary artery rupture.

d. Rationale: When a pulmonary artery pressure tracing indicates a wedged waveform when the balloon is deflated, this indicates that the catheter has advanced and has become spontaneously wedged. If the catheter is not repositioned immediately, a pulmonary infarction or a rupture of a pulmonary artery may occur. If the catheter is becoming occluded, the pressure tracing becomes blunted, and pulmonary edema and increased pulmonary congestion increase the pulmonary artery waveform. Balloon leaks found when injected air does not flow back into the syringe do not alter waveforms.

The nurse notes premature ventricular contractions (PVCs) while suctioning a patient's endotracheal tube. Which next action by the nurse is indicated? a. Plan to suction the patient more frequently. b. Decrease the suction pressure to 80 mm Hg. c. Give antidysrhythmic medications per protocol. d. Stop and ventilate the patient with 100% oxygen.

d. Stop and ventilate the patient with 100% oxygen. Dysrhythmias during suctioning may indicate hypoxemia or sympathetic nervous system stimulation. The nurse should stop suctioning and ventilate the patient with 100% O2. There is no indication that more frequent suctioning is needed. Lowering the suction pressure will decrease the effectiveness of suctioning without improving the hypoxemia. Because the PVCs occurred during suctioning, there is no need for antidysrhythmic medications (which may have adverse effects) unless they recur when the suctioning is stopped and patient is well oxygenated.

The nurse educator is evaluating the care that a new registered nurse (RN) provides to a patient receiving mechanical ventilation. Which action by the new RN indicates the need for more education? a. The RN increases the FIO2 to 100% before suctioning. b. The RN secures a bite block in place using adhesive tape. c. The RN asks for assistance to resecure the endotracheal tube. d. The RN positions the patient with the head of bed at 10 degrees.

d. The RN positions the patient with the head of bed at 10 degrees. The head of the patient's bed should be positioned at 30 to 45 degrees to prevent ventilator-associated pneumonia. The other actions by the new RN are appropriate.

The nurse is caring for a patient with a subarachnoid hemorrhage who is intubated and placed on a mechanical ventilator with 10 cm H2O of peak end-expiratory pressure (PEEP). When monitoring the patient, the nurse will need to notify the health care provider immediately if the patient develops a. O2 saturation of 93%. b. green nasogastric tube drainage. c. respirations of 20 breaths/minute. d. increased jugular venous distention.

d. increased jugular venous distention. Increases in jugular venous distention in a patient with a subarachnoid hemorrhage may indicate an increase in intracranial pressure (ICP) and that the PEEP setting is too high for this patient. A respiratory rate of 20, O2 saturation of 93%, and green nasogastric tube drainage are within normal limits.

To maintain proper cuff pressure of an endotracheal tube (ET) when the patient is on mechanical ventilation, the nurse should a. inflate the cuff with a minimum of 10 mL of air. b. inflate the cuff until the pilot balloon is firm on palpation. c. inject air into the cuff until a manometer shows 15 mm Hg pressure. d. inject air into the cuff until a slight leak is heard only at peak inflation.

d. inject air into the cuff until a slight leak is heard only at peak inflation. The minimal occluding volume technique involves injecting air into the cuff until an air leak is present only at peak inflation. The volume to inflate the cuff varies with the ET and the patient's size. Cuff pressure should be maintained at 20 to 25 mm Hg. An accurate assessment of cuff pressure cannot be obtained by palpating the pilot balloon.


Ensembles d'études connexes

Microeconomics 231 - Test 2 - Practice Questions

View Set

Chapter 36 Abdominal Trauma - Study Set

View Set

Finding a Constant of Proportionality

View Set

Handout 9-Reproduction of Flowering Plants

View Set

development part 1 practice questions

View Set

Chapter 43: Drugs Affecting Blood Pressure, 47: Lipid-Lowering Agents, & 48: Drugs Affecting Blood Coagulation

View Set